You are on page 1of 76

Lecture: Proteoglycans and Glycoproteins

1. Describe the general structure of proteoglycans


a. Except for Hyaluronic acid all other GAGs are attached to protein
Proteoglycan monomers. Each chain contains greater than 100
monosaccharides and structure resembles a bottle brush.
2. Indicate what sugar derivatives are used for GAG synthesis
a. D-Glucosamine and D-Galactosamine (They are acetylated) are the
sugar derivatives. The acid derivatives are D-Glucoronic Acid and LIduronic Acid. (Keratan Sulfate is the exception and has D-Galactose
instead of an acid sugar)
3. Discuss the general composition of GAGs and the possible bottlebrush structure of proteoglycans
a. GAGs are compressible and most are extracellular. They maintain
cell shape, adhesion, migration, cell-cell communication. Made up of
repeating disaccharide unit: [acidic sugar amino sugar]n
4. Describe the synthesis of hyaluronic acid and the extracellular
assembly of proteoglycan aggregates using link proteins leading
to shock absorbing aggregates
a. Carbohydrate and protein are linked through a trihexoside (Gal-GalXyl) and a Serine Residue. This carbohydrate-Protein complex then
associates with a molecule of hyalournoic acid with the aid of small
proteins called link proteins.
5. Discuss the function of hyaluronic acid related to facilitation of
cell migration
a. Hyaluronic acid has anti-adhesive properties and therefore might be
useful in postsurgical wound-healing as well
6. Discuss the structure and function of heparin
a. Heparin is made up of D-Glucoronic/L-Iduronic Acid and
Glucosamine. It functions as an inhibitor of blood clotting. Heparin
induces the release of cell surface-associated TFPI.
7. Discuss some of the roles of glycoproteins (glycocalyx, blood
proteins, mucins)
a. O-linked oligosaccharides determine ABO blood group determinants
b. Mucins are large proteins with negatively charges sialic acid, Nacetyl neuraminic acid (NANA) occupy a large space, trap water and
serve as protective barriers.
c. Glycocalyx- cell-surface recognition (receptors) and cell surface
antigenicity.
8. Outline the synthesis of O-linked glycoproteins and indicate the
amino acid side chains involved
a. OH group of Serine or Threonine linked by N-acetylgalactosamine
b. Protein is synthesized in rER and then extruded into lumen of ER.
Glycosylation
occurs
in
the
golgi
through
action
of
glycosyltransferases that add sugars one at a time. Glycosylation
begins with transfer of N-acetylgalactosamine to Serine or
1

Threonine OH group. After glycosylation the glycoprotein moves


through the golgi for packaging.
9. Outline the synthesis of N-linked glycoproteins and indicate the
significance of dolichol-pyrophosphate and mannose involvement
(see Fig.14.16 Lippincott p. 168).
a. Amide grounp of Asparagine
b. N-linked are also synthesized in ER and Golgi. Uses a lipid dolichol
and phosphorylated lipid (dolichol phosphate).
c. The protein is made in the rER and enters the lumen. Sugars are
added to the dolichol pyrophosphate by membrane bound
glycosyltransferases, the sugars on the dolichol pyrophosphate are
then transferred onto the polypeptide chain by proteinoligosaccharide and linked to protein via an asparagine. Chains are
completed in the golgi either as complex (various sugars added) or
high mannose.
10. Describe the differences between the synthesis of O-linked and
N-linked glycoproteins
a. O-linked adds sugars one by one to Serine and Threonine. In Nlinked the sugars are added to the Dolichol pyrophosphate and then
to an Asparagine on the protein.
Lecture: Lysosomal Storage Disorders
1. Describe
the
general
lysosomal
degradation
of
GAGs,
glycoproteins, and sphingolipids
a. The lysosomal enzymes remove one sugar unit at a time from these
molecules.
2. Outline the causes of lysosomal storage disorders in general
a. A specific enzyme of the degradative pathway is deficient in these
disorders, resulting in accumulation of the substrate of the pathway.
The rate of biosynthesis of the compound is usually normal, but the
rate of degradation is slow due to inherited deficiency of the
enzyme. The substrate of the deficient enzyme accumulates in the
lysosomes, resulting in swelling of the lysosomes.
3. Describe the biochemical basis of symptoms of patients with
lysosomal storage disorders (hepatosplenomegaly, mental
retardation, macular cherry red spot)
a. Hepatosplenomegaly- Storage of insoluble intermediates in the
mononuclear phagocyte system
4. Describe the symptoms and biochemical basis of Hunters and
Hurlers syndrome; include the deficient enzyme and indicate the
products that accumulate in the two disorders.
a. Deficiency in the breakdown of GAGs. Characterized by the
accumulation of dermatan sulfate and heparan sulfate. In Hurlers
syndrome deficient enzyme is Iduronidase. Urine is positive for

5.

6.

7.

8.

9.

GaGs and you have corneal clouding. Hunters syndrome is milder


form and has a deficient Iduronate Sulfatase. NO corneal clouding.
Describe Tay-Sachsdisease and relate to the deficient enzyme,
accumulating
compound, (onion-shell inclusions) and the
biochemical basis for the cherry-red macula
a. Deficient enzyme is -Hexoaminidase A and the accumulating
substrate is Ganglioside (GM2). Onion shell inclusions in lysosomes
and Gangliosides are responsible for the cherry red spot.
Describe Fabrys disease and relate to the deficient enzyme,
accumulating compound, and physical appearance of patients
(dark reddish skin lesions with bathing trunk distribution,
impaired sweating, muscle weakness)
a. X-linked recessive disorder and the deficient enzyme is Galactosidase and the accumulating substrate is Globoside
(ceramide trihexoside).
Describe Gauchers disease and relate to the deficient enzyme,
accumulating compound, general cell appearance (crumpled
tissue paper); discuss the infantile and adult forms of Gauchers
disease
a. Most common lysosomal storage disorder. The deficient enzyme is
-Glucosidase and the accumulating substrate is Glucosyl ceramide
(Glucocerebroside). The adult form is most common and shows
marked hepatosplenomegaly and osteoporosis of long bones. The
crumples tissue paper appearance of the cytoplasm of Gaucher cells
is caused by enlarged elongated lysosomes filled with
glucocerebrosides.
Describe Niemann-Pick disease and relate to the deficient
enzyme, accumulating compound, general cell appearance
(Foamy appearing cells); discuss Type A and Type B
a. Niemann-Pick disease is a deficiency of Sphingomyelinase and the
accumulating substrate is sphingomyelin. Type A is severe infantile
form and Type B appears later in childhood and presents with
hepatosplenomegaly. Cherry-red spot in macula on retinal
examination. Deficiency of sphingomyelinase causes lipid droplet
accumulation- Foamy cell Apearance
Discuss the role of the mannose-6-P marker for transport of
lysosomal enzymes into lysosomes. Indicate the biochemical
defect in I-Cell disease
a. Enzymes synthesized in the ER are transported to the Golgi
apparatus. In the Golgi there is phosphorylation of mannose to form
mannose 6-phosphate. Enzymes that have this Mannose 6phosphate marker are transported to the lysosomes. Enzymes
without the marker are secreted by the cell. I-cell disease is when
the golgi doesnt add this marker leading to an increase of
lysosomal enzymes in the blood. Accumulation of GAGs and
Sphingolipids in the lysosomes. Presence of intracytoplasmic
3

inclusions in fibroblasts and these cells are called I-cells.


Symptoms similar to Hurler but more severe and earlier age of
onset.
Lectures: Oxidation of fatty acids & ketogenesis (2 lectures)
1. Describe adipose tissue lipolysis and the regulation of hormone
sensitive lipase. Indicate mechanism of fatty acid transport to the
tissues
a. HS Lipase is inhibited by insulin in a well fed state and stimulated by
epinephrine. It cleaves TAGs to free FA in the adipose tissue. The
Free FA then are transported to the Liver and Skeletal/Cardiac
muscle by Albumin. In these tissues the Free FA will undergo Oxidation. Fatty acids are NOT used by the brain.
2. Describe fatty acid activation
a. Fatty Acids are activated to fatty acyl CoA by fatty acyl CoA
synthetase (Thiokinase). This enzyme is present in the outer
mitochondrial membrane.
3. Describe the carnitine shuttle mechanism.
a. Acyl CoA cannot traverse the inner mitochondrial membrane so
carnitine binds to the acyl group to form acyl-carnitine that is
transported across the inner mitochondrial membrane via
translocase (When this happens a carnitine comes from the matrix
into the intermembrane space). Then CPT II in the matrix takes of
the carnitine and the Acyl CoA is used for -oxidation. CPT I is
inhibited by malonyl CoA (Formed during FA synthesis by Acetyl CoA
Carboxylase (ACC))
4. Explain the biochemical consequences of defects in transport of
fatty acids (carnitine deficiency and CPT deficiency)
a. Carnitine Deficiency- Transport of long chain fatty acids into the
mitochondria is impaired and -oxidation is decreased. You get
hypoglycaemia due to impaired gluconeogenesis. (Acetyl CoA is an
activator of pyruvate carboxylase). Ketogenesis is decreased if liver
carnitine is deficient. Myopathic carnitine deficiency afects just
muscles.
b. CPT I- characterized by a hypoclycemia and predominantly afects
the liver
c. CPT II- Characterized by cardiomyopathy and muscle weakness
(myopathic form). Lipid deposits are found in skeletal muscle.
Prolonged exercise results in myoglobinuria and elevated CK levels.
5. List the reactions of the mitochondrial -oxidation pathway (not
substrate names)
a. Acyl CoA Dehydrogenase (FAD), Enoyl CoA reductase, 3-Acyl CoA
Dehydrogenase (NAD+), Thiolase (Cleaves)
6. Outline the energetics of -oxidation
a. Palmitoyl CoA (16 C), produce 8 Acetyl CoA, 7 NADH and 7 FADH2
4

7. Indicate the end products of oxidation of odd chain fatty acids


a. Products are Acetyl CoA until the end where you end up with a
Propionyl CoA. This is converted to Succinyl CoA to enter the TCA
cycle. Converted by Propionyl CoA carboxylase and Methylmalonyl
CoA mutase
8. Outline oxidation of branched chain fatty acids and indicate the
biochemical defect in Refsums disease.
a. -Oxidation of branched chain fatty acids takes place in the
peroxisomes. Refsum disease is a disorder characterized by
deficiency of the peroxisomal phytanyl CoA -hydroxylase.
Phytanate accumulates in tissues, especially the neurologic tissues.
Characterized by visual defects, ataxia and skeletal manifestations.
Management includes dietary restriction of branched chain FA.
9. Distinguish between medium-chain acyl-CoA dehydrogenase
deficiency and Jamaican vomiting sickness based on the
pathogenetic mechanism and biochemical alterations
a. MCAD- Decreased -Oxidation of medium chain fatty acids. C6-C10
carnitines in blood. Increased flux through -oxidation (Dicarboxylic
Acids in Urine). Hypoglycemia because of decreased utilization of
fatty acids by peripheral tissues. Increased reliance on glucose as
an energy source. Decreased ATp and acetyl CoA to activate
gluconeogenesis. Hypoketonemia because of decreased -oxidation
in the liver and decreased substrate for ketogenesis (Acetyl CoA).
Presence of CK-MB and myoglobin in Urine. Most common inherited
autosomal recessive enzyme disorder.
b. Jamaican Vomiting- Unripe Ackee fruit contains hypoglycin A which
inhibits MCAD
10. Discuss Zellwegers syndrome as a disorder of peroxisomes
a. Very long chain fatty acids (22 to 26 Carbon atoms) are initially
oxidized in the peroxisomes. The shortened fatty acid is transported
to mitochondria for further oxidation. Zellweger syndrome is
defective peroxisomal biogenesis afecting the liver and brain.
Levels of C 26 fatty acids in circulation are increased in Zellweger
syndrome. Hepatomegaly and neurological manifestations are
associated. Fatal.
11. Describe the pathway of hepatic ketogenesis and list the
ketone bodies
a. After -oxidation the products of Acetyl CoA under go ketogenesis in
the liver. This replenishes the NAD+ needed by 3 Hydroxyacyl CoA
Dehydrogenase for -Oxidation. The Acetyl CoA is acted upon by
Thiolase, HMG CoA synthase, HMG CoA lyase and 3Hydroxybutyrate Dehydrogenase. Acetone and 3-Hydroxy Butyrate
are the ketone bodies that are formed. (Process happens in liver
mitochondria). Acetone is lost via the lungs giving a fruity odor.
12. Explain ketone body utilization in peripheral tissues

a. Ketone bodies are formed in the liver and used in the peripheral
tissues. 3-Hydroxybutyrate is oxidized to acetoacetate and
acetoacetate is activated to acetoacetyl CoA by Thiophorase.
Thiolase then cleaves to form 2 Acetyl CoA for Krebs.
13. Prepare a concept map indicating the steps involved in the
generation of ketosis in starvation and uncontrolled type 1
diabetes mellitus.
a. There is increased Ketogenesis during starvation because Acetyl
CoA stimulates gluconeogenesis. Also the increased NADH/NAD+
ratio results in formation of malate from oxaloacetate for
gluconeogenesis. Acetyl CoA is then used for ketogenesis rather
than krebs.
b. Ketoacidosis in Diabetes Mellitus- In uncontrolled diabetes mellitus
there is no insulin so there is a lot of lipolysis and uncontrolled
production of ketone bodies. The production is greater than the
utilization so you find ketone bodies in the urine. Ketone bodies are
weak acids and lose protons resulting in acidosis. You get acetone in
breath causing the fruity odor and rate of respiration increases to
compensate for metabolic acidosis.
14. Correlate laboratory data in ketoacidosis (laboratory data in
blood and urine) to the clinical signs in the patient; include
hyperventilation
Lecture: Fatty acid synthesis
1. Indicate the main sites in the body and the metabolic condition
under which fatty acid de novo synthesis takes place
a. Major sites of Fatty Acid synthesis is the Liver and lactating
mammary gland. Minor sites are adipose tissue and kidney. Most
likely to occur in the well-fed state. This takes place in the cell
cytoplasm.
2. Outline the actions of citrate lyase and the malic enzyme in the
liver.
a. Acetyl CoA is converted to Citrate for transport out of the
mitochondria. Citrate Lyase is in the cytosol and converts the citrate
back into OAA and Acetyl CoA. Malic Enzyme is in the cytosol and
converts Malate (from the OAA by product of Citrate Lyase in the
cytosol) to Pyruvate which is transported into the matrix to create
Acertyl CoA. Malic enzyme also forms an NADPH.
3. Explain how acetyl-CoA carboxylase synthesizes malonyl-CoA and
how the reaction is regulated (short-term and long-term)
a. First committed step of FA synthesis is catalyzed by Acetyl CoA
carboxylase (ACC). RLE for FA synthesis. Needs biotin and adds a
CO2 to the methyl end of acetyl CoA. This reaction requires 1 ATP
per malonyl-CoA formed (inhibits CPT).

4.

5.
6.

7.

b. Short Term Regulation- Citrate activates ACC by shifting it to the


polymer. Palmitoyl CoA shifts the equilibrium towards the monomer,
inhibiting ACC. Glucangon, Epinephrine, and norepinephrine all shift
the
equilibrium
towards
the
monomer
inhibiting
ACC
(phosphorylating).
c. Long Term Regulation- High carbohydrate and fat-free diets lead to
increased synthesis of ACC resulting in increased synthesis of FA.
High-Fat diets, fasting and glucagon lead to decreased synthesis
resulting in decreased synthesis of FA.
Outline the fatty acid synthetic pathway and indicate the role of
the acyl carrier protein in the pathway
a. The growing fatty acyl-chain on ACP makes it available for the
enzyme activities of the opposite part of the dimer. If the fatty
acyl-groups would be released each time, then they would have to
be activated to fatty acyl-CoA which needs ATP.
b. Synthesis of Palmitate (16 C) from acetyl CoA requires 14 NADPHs,
7ATPs. Fatty Acid Synthase does all seven steps of synthesis. It links
malonyl CoAs together until the final product is formed. Each time 2
NADPH are used and 1 ATP for the Malonyl CoA formation.
Identify the sources of NADPH for fatty acid synthesis
a. Pentose Phosphate Pathway, and reaction by Malic Enzyme, Malate
to Pyruvate in the cytosol.
Discuss malonyl-CoA inhibition of carnitine-palmitoyl transferase
I (CPT I)
a. Malonyl-CoA inhibits CPT I to make sure that Fatty Acid degradation
does not take place at the same time as FA synthesis.
Outline the elongation and desaturation of fatty acids in humans.
Explain the conversion of linoleic acid to arachidonic acid and linolenic acid to docosahexaenoic acid
a. Elongation occurs primarily in the membranes of the ER and
involves the addition of malonyl CoAs to palmitate. In the
mitochondria (minor pathway) FA elongation occurs by a reversal of
-Oxidation.
b. Fatty Acid Desaturation- Sites of unsaturation are introduced by
fatty acyl-CoA desaturases that introduce double bonds at positions
5,6,9. Linolenic and linoic acid have double bonds past this point
and thats why they are essential.
c. -Linolenic Acid Docosahexaenoic Acid (PUFA).

8. List the major tissues of triacylglycerol synthesis and storage


a. Liver synthesizes TAGs and Adipose Tissue Stores them.
9. Distinguish between the glycerol-3-P pathway and the MAG
pathway for triacylglycerol synthesis
a. Glycerol 3-P is formed from DHAP which is branching out of glycolysis. In the liver
it can also be formed from free glycerol by glycerol kinase. Usage of two fatty acyl7

CoAs leads to phosphatidic acid which is intermediate for TAG synthesis in fat cells
and liver. It is not intermediate in the MAG pathway in intestinal mucosal cells. TAGs
are eventually formed from phosphatidic acid by cleavage of the phosphate leading
to DAG. Usage of another fatty acyl-CoA forms TAG. (3 FA CoAs used)
b. Intestinal mucosal cells can start TAG synthesis with dietary MAG which is formed
by pancreatic lipase in the lumen of the intestines. The usage of two fatty acyl-CoAs
leads directly to TAG without phosphatidic acid as intermediate. The purpose of
pancreatic lipase is to form a molecule that can be taken up into the intestinal cells
and it cleaves TAG to MAG but not further on.
Lecture: Eicosanoids
1. Describe in general the formation of eicosanoids including
prostaglandins, thromboxanes and leukotrienes
a. Phospholipase A2 cleaves PIP2 to generate Arachidonic Acid and
Lyso-PIP2. The Arachidonic Acid is then worked on by 5Lipoxygenase to form Leukotrienes or used by COX to form
Thromboxanes and Prostaglandins. COX is rate limiting for
Prostaglandin Synthesis. COX forms PGG2 which then goes on to
form PGH2 which gives rise to all the thromboxanes and
prostaglandins.
2. Discuss the role of cortisol in eicosanoid synthesis
a. Cortisol inhibits synthesis of all eicosanoids because it doesnt allow
formation of arachidonic acid by phospholipase A2
3. Outline the cyclic and linear pathways starting with arachidonic
acid. (Look in Notebook)
a. Cyclic- COX pathway
b. Linear- Leukotriene Pathway
4. Describe the general effects of PGI2 versus TXA2, and PGE2 versus
PGF2
a. PGI2- Prostacyclin and is produced by endothelium of vessels.
Vasodilation, inhibits platelet aggregation and increases cAMP
b. TXA2- Thromboxane and is produced by platelets. Promotes platelet
aggregation, vasoconstriction, mobilizes intracellular calcium,
contraction of smooth muscle, bronchoconstriction and decrease in
cAMP
c. PGE2- Vasodilation, relaxation of smooth muscle
d. PGF2Vasoconstriction,
contraction
of
smooth
muscle,
bronchoconstriction.
5. Describe the two enzyme activities of prostaglandin H synthase
and cyclooxygenase (COX)
a. COX 1- A constitutive enzyme found in almost all tissues. Involved
with normal physiological functions of Prostacyclin and
Thromboxane. Gastric protection and limiting acid secretion,
maintenance of renal blood flow, vascular homeostasis and
hemostasis.

b. COX 2- Non-constitutive and can be induced in a limited number of


tissues in response to immune and inflammatory mediators (TNF,
cytokines, tumor promoters, endotoxin). Expression of COX-2 is
inhibited by glucocorticoids. Induction of COX-2 leads to increased
prostaglandin synthesis which results in pain, heat, redness,
swelling and fever.
c. Prostaglandin H synthase has cyclooxygenase and peroxidase
activities.
6. Differentiate between COX-1 and COX-2 and discuss their
inhibition by NSAIDs
a. NSAIDs, like ibuprofen and acetaminophen inhibit COX-1 and COX-2
as competitive reversible inhibitors
7. Indicate the eicosanoids formed from EPA and their biomedical
activity compared to series-2 eicosanoids.
a. Eicosanoids formed from EPA (fish oil) are considered series 3. This
forms PGI3 and TXA3. When Eicosanoids are formed from Arachidonic
acid it is called the 2 series. PGI2=PGI3 in potency. However, TXA2 >
TXA3 in potency. Diet rich in fish oil will favour prevention of clotting
because youll have TXA3
8. Describe the synthesis of leukotriene LTA and the following
change to LTB and to LTC
a. Arachidonic acid is worked on by 5-Lipoxygenase and you get LTA.
LTA is used in mast cells to form LTC by addition of GSH. LTB is the
other product of LTA and is a chemoattractant and WBC adhesion
helper.
9. Indicate the synthesis of components and actions of the slowreacting substance of anaphylaxis (SRS-A)
a. LTC is cleaved to LTD and LTE. They contain a cysteine that
originated in GSH. These 3 are components of Slow-reacting
substance. They are much more potent than histamine. Increase
vascular
permeability
but
severe
bronchoconstriction.
Vasoconstriction and lung edema
10. Discuss the biochemical basis for using corticoids, LOXinhibitors or inhibitors of CysLT receptors in the management of
asthma.
a. Corticoids would inhibit the formation of all Eicosonoids. LOX
inhibitors would only afect the Leukotrienes. CysLT would afect the
receptors for Leukotrienes so they would not be able to produce
their efect through G-proteins. (All Eicosanoids work through GProteins.
Lecture: Cholesterol (27 C) metabolism
1. List the functions of cholesterol in the human body
a. Component of Cell Membranes, Precursor of bile acids and precursor
of steroid hormones and Vitamin D.
9

2. Outline cholesterol synthesis and discuss the branch-point at the


level of farnesyl-PP
a. Synthesis requires acetyl-CoA, NADPH, and ATP. RLE is HMG CoA
reductase. Occurs in the cytoplasm with enzymes in cytosol and ER.
IPP (5 C) DPP (5 C) GPP (10 C) FPP (15 C) Squalene (30 C)
Lanesterol (30 C) Cholesterol (27 C)
b. FPP can form dolichol, ubiquinone and protein prenylation
3. Indicate the biochemical basis of Smith-Lemli-Opitz syndrome
(defect in the cholesterol biosynthetic pathway)
a. A genetic defect of 7-Dehydrocholesterol Reductase. Doesnt let you
form the double bond in ring B. Leads to microencephaly and low IQ.
4. Explain how the pathway is regulated at the level of HMG-CoA
reductase
a. Cholesterol is a feedback inhibitor. Low cholesterol stimulates the
release of a regulator (SREBP) protein form the ER. SREBP binds to a
region in the HMG CoA reductase gene called sterol responsive
element (SRE) resulting in increased transcription of the HMG CoA
reductase gene. SREBP is cleaved by SCAP.
b. High intracellular concentrations of AMP stimulate AMP kinase which
phosphorylates HMG-CoA and inactivates it.
c. Statin Enzymes cause upregulation (reversible inhibitors of HMG
CoA reductase). This makes the cell produce more LDL receptors
that take in LDL so they can get more cholesterol. This efectively
limits the cholesterol in the serum.
d. Glucagon Phosphorylates and inactivates HMG CoA reductase
e. Ubiquitin- high concentration of cholesterol and mevalonate leads to
degradation of the enzyme.
5. Outline bile acid synthesis and its regulation; indicate the rate
limiting step in bile acid synthesis
a. Rate limiting step in bile acid synthesis is 7--hydroxylase. It is
inhibited by cholic acid. Cholic acid and Chenodeoxycholic acid are
the Primary bile acids. Cholesterol is hydroxylated, double bond is
reduced, and is shortened by 3 carbons.
6. Describe the conjugation of primary and secondary bile salts in
the liver and its significance
a. Bile acids are conjugated with glycine or taurine before they leave
the liver. Action of intestinal bacteria on bile salts convert primary
bile salts into secondary bile salts by removing hydroxyl groups.
These prevent Cholelithiases (precipitation of Cholesterol).
7. Discuss bile composition and predict biochemical reasons for
gallstone formation
a. Bile is made of up of cholesterol, PC (Lecithin) and an amino acid.
Deficiency of Lecithin and/or bile salts will lead to cholesterol
precipitating in the gall bladder.
8. Describe the biochemical basis for use of chenodeoxycholic acid
for management of cholelithiasis
10

a. Chenodeoxycholic acid is the after the step of formation of cholic


acid and does not feed-back inhibit bile acid synthesis (7-hydroxylase). With that it can lead itself to formation of cholic acid
without inhibiting anything. You will have more bile salts to increase
solubility of cholesterol.
Lecture: Steroid hormones
1. Name the five classes of steroid hormones (glucocorticoids,
mineralocorticoids, androgens, estrogens and progestins)
a. Cortisol
(Glucocorticoid),
Corticosterone
and
Aldosterone
(Mineralocorticoids), Testosterone (Androgen), Estradiol (Estrogens)
and Progesterone (Progestins)
2. Discuss steroid hormone synthesis related to STAR and
desmolase (CYP11A)
a. Cholesterol is used for steroid hormone synthesis and is taken up
via LDL, HDL or it is synthesized. The transport of cholesterol into
the mitochondria is performed by STAR. The rate limiting step in
steroid hormone synthesis is performed by desmolase and it is the
cholesterol side chain cleavage enzyme (CYP 11). Needs NADPH and
O2
3. Describe the steroid hormones synthesized and released from the
adrenal cortex
(specified for zona fasciculata, z. glomerulosa
and z. reticularis)
a. Zona Fasciculata- Cortisol (Stimulated by ACTH)
i.
Dominant glucocorticoid found in humans. Involved in stress
adaptation, and needed with glucagon in liver for
gluconeogenesis. Leads to a degradation of muscle protein. It
also reduces inflammation by inhibiting Phopholipase A 2. Only
unbound cortisol can enter the cells. In areas of inflammation
Cortisol unbinds Transcortin and enters cells stopping the
release of Arachidonic Acid
b. Zona
GlomerulosaAldosterone,
Androgens
(DHEA,
Androstenedione)
ii.
Stimulates renal reabsorption of Na + and excretion of K+.
Aldosterone is the principal mineralocorticoid and raises blood
pressure and fluid volume. Its release is stimulated by the
hormone angiotensin II formed by ACE.
c. Z. Reticularis- Release Androgens (DHEA, Androstenedione)
iii.
Mainly produce DHEA and adrostenedione but also some
testosterone. The weak androgens are changed in fat cells to
estradiol and in peripheral tissue to testosterone.
4. Indicate the synthesis and the metabolic actions of cortisol
a. Synthesized from Cholesterol. The RLE is Desmolase which forms
the product of Pregnenolone and is a precursor to Cortisol.
Metabolic Actions are listed above.
11

5. Indicate the synthesis and the metabolic actions of aldosterone


a. Same as above except Pregnenolone goes to Corticosterone and
then Aldosterone.
6. Indicate the synthesis of DHEA and testosterone in the adrenal
cortex
a. DHEA is precursor of testosterone and LH receptor in Leydig cells
increases c-AMP and PKA resulting in testosterone synthesis.
7. Describe the synthesis of testosterone in testes and of estradiol
in ovaries
a. Estradiol is formed from Testosterone (Testosterone is formed from
DHEA). Stimulated by FSH and controls the Menstrual cycle and
promotes the development of secondary female characteristics.
8. Discuss congenital adrenal hyperplasias (CAH) and describe
clinical features and biochemical basis of manifestations in
patients with CYP 21 and CYP 11B and CYP 17 deficiencies
a. CYP 21/CYP 11- Cortisol cannot be formed so release of ACTh is not
inhibited. ACTH stimulates adrenal cortex and more androgens are
formed in Z. Reticularis as their synthesis doesnt need CYP 21 or
CYP11B. This leads to masculinzation of female fetuses/patients.
CYP 21 is the most common form of CAH.
b. CYP 17- phenotypically female but cant mature. CYP 17 is needed
for DHEA synthesis and of testosterone. Deficiency leads to female
phenotype. Estreogen synthesis in ovaries starts with cholesterol
and needs testosterone as an intermediate. CYP 17 leads to females
that are unable to mature. Aldosterone synthesis is increased in CYP
17 deficiency because there is uncontrolled stimulation of the
adrenal cortex (ACTH isnt inhibited).
c. Additional 3--Hydroxysteroid Dehydrogenase Deficiency- Virtually
no Steroids. Patients have female like genitalia.
9. Indicate the hormone abnormality in Cushings and Addisons
disease
a. Cushings syndrome- High Cortisol concentration and low ACTH in
plasma. Excess protein loss and to fat distribution. (Caused by
hyperfunction normally due to tumor)
b. Addisons is high ACTH low Cortisol and aldosterone in plasma.
Leads to muscle weakness, fatigue etc. (Caused by adrenal cortex
atrophy due to disease, hypofunction of adrenal cortex)
10. Indicate the biochemical basis for the occurrence of
hyperglycemia in Cushings syndrome
a. High Cortisol means more glucose via gluconeogenesis. Constant
high levels of cortisol leads to hyperglycemia. Cortisol also leads to
the release of epinephrine which inhibits insulin release from the cells and stimulates release of glucagon from -cells of pancreas.
Cushings syndrome is characterized by increased release of glucose
form the liver into the blood and by decreased uptake of glucose

12

into muscle and fat cells due to lack of insulin (GLUT-4 will not take
in glucose). More blood in plasma.
Lectures: Lipoproteins I & II (2 lectures)
1. Discuss lipid transport in the blood by lipoproteins and albumin.
a. Albumin transports lipids but not cholesterol. Transports free fatty
acids, bilirubin or other lipids in its hydrophobic pockets. Lipids are
non-covalently bound
b. Lipoproteins are macromolecular complexes of a phospholipid
monolayer with mainly PC. Transport non-polar lipids (TAGs and CE)
inside in serum. Specific proteins (apoproteins) are needed for
lipoprotein functions.
2. Describe the location and function of lipoprotein lipase (heart,
skeletal muscle and adipose tissue)
a. LPL is extracellular bound to the capillary walls of heart, skeletal
muscle and adipose tissue. It cleaves TAGs in chylomicrons and
VLDL which are TAG rich. LPL needs apo C=II for activation. The
generated free fatty acids can be used for -oxidation in the
heart/muscle or stored in fat cells as TAGs or transported in blood
by albumin. The generated free glycerol is taken up by the liver and
can be used as glycerol backbone for TAG synthesis at insulin ruling.
b. Heart LPL has a smaller Km for TAGs. The heart needs fatty acids for
energy metabolism even when plasma lipoprotein concentration is
low.
c. Adipose LPL has a large Km and acts at elevated lipoprotein
concentrations. It is activated by insulin [high blood glucose levels].
Fatty acids are stored as TAGs in fat cells.
3. Discuss chylomicron metabolism starting from formation to its
uptake into the liver
a. CM release from the liver needs apo B-48. CMs join the blood
circulation at the thoracic duct. For the TAGs in CM to be cleaved
the CM needs to have apo C-II (LPL needs apo C-II to cleave). CM
gets both apo C-II and apo E from HDL in blood. Once the TAGs have
been cleaved the big CM become smaller remnants that contain
dietary CE. The apo C-II is then given back to HDL. The CM
remnants are taken up by the liver and still contain the apo B-48
and apo E.
4. Discuss the transport of dietary triacylglycerols from the
intestine to the peripheral tissues
a. TAGs are transported to peripheral tissues by CM and VLDL
5. Describe the conversion of VLDLs into IDLs by lipoprotein lipase
and the uptake of IDL by the liver
a. Apo B-100 is needed for the release of VLDL from the liver. VLDL
gets apo C-II and apo E from HDLs in blood. LPL cleaves TAGs in
VLDLs and the remnants of VLDLs are formed and apo C-II is given
13

back to HDL. This VLDL remnant is called IDL. 50% of IDL is taken up
into the liver via remnant receptors which recognize apo E in IDL,
via LDL-receptor protein (LRP), or via LDL-receptor (apo B-100/apo
E)
b. The remaining IDLs in the blood will be used to form LDLs. Hepatic
lipase (HTGL) cleaves TAGs in IDLs. Apo E is given back to HDLs and
the formed LDLs have only apo B-100. 70% of LDLs are taken into
the liver by LDL-receptors which recognize apo B-100. LDLs are also
taken up by cells which contain LDL receptors in their plasma
membrane as they are in the need for cholesterol. (receptor
mediated endocytosis)
i. After a cholesterol rich meal the CM remnants lead to high
free cholesterol levels in cytosol of hepatocytes reducing
LDL-receptor synthesis leading to increased levels of serum
LDL.
6. Describe the conversion of IDLs to LDLs performed by hepatic
TAG lipase
7. Discuss the uptake of LDL into the liver and extra-hepatic tissues
by receptor mediated endocytosis
8. Describe the regulation of LDL-receptor synthesis
a. At high free cholesterol levels the cytosolic enxyme ACAT is
activated and a reservoir of cholesteryl esters is formed. If there is
still free cholesterol left in cytosol then the synthesis of HMG-CoA
reductase and also synthesis of LDL receptors are down-regulated
(inhibition of the activation of SREBP which would take place a low
free cytosolic cholesterol levels).
9. Outline the receptor-mediated process involving LDL and LRP
receptors
a. LRP receptors need both apo E and apo B-100 for uptake (IDL has
both)
b. LDL Receptors need apo B-100 for uptake (LDL has this because it
gave its apo E to HDL)
10. Discuss the functions and effects of deficiency of the main
apolipoproteins (apo B-48, apo B-100, apo C-II, apo E and apo A1)
a. Apo B-48- release of CM from liver, lipoprotein receptor recognition
i. Synthesized in intestinal mucosal cells
b. Apo B-100- release of VLDL from liver, lipoprotein receptor
recognition
i. Synthesized in liver cells and is largest polypeptide chain
known
ii. Needed for the uptake of LDL by LDL receptors of liver or
extra hepatic tissue
c. Apo C-II- HDL apoprotein donation to CM or VLDL, activation of LPL
d. Apo E- lipoprotein receptor recognition, HDL apoprotein donation to
CM or VLDL
14

i. Needed for CM remnants uptake into liver by remnant


receptors. Also needed for uptake of IDL by LRP.
e. Apo A-1- Activation of LCAT (PCAT)
11. Describe the reverse cholesterol transport performed by HDL
a. HDLs contain apo A-1 for activation of LCAT which forms in blood CE
using as substrates free cholesterol (provided by ABC transporter in
plasma membrane) and PC of HDL. HDL delivers these CE to the
liver. HDLs can be formed in blood by addition of phospholipids to
apo A-1.
12. Describe the synthesis, location and action of lecithin:
cholesterol acyltransferase (LCAT)
a. This enzyme is synthesized by the liver and released into the blood
where it is activated by apo A-1. LCAT forms cholesteryl esters in
blood. These CE are stored inside the HDL.
13. Describe biochemical basis of atherosclerosis in Tangier
disease and name the defective transporter
a. Tangier disease is acquired and related to obesity, smoking, medical
drugs and cholesterol reducing drugs. Very low serum HDL in
childhood. It is related to a defective cholesterol ABC transporter in
the plasma membrane. This leads to less substrate for LCAT and to
early degradation of lipid poor apo A-1 in serum and very low HDL.
Orange tonsils, and hepatosplenomegaly.
14. Describe the function of cholesterol ester transfer protein
(CETP)
a. CETP is the cholesteryl ester transfer protein and allows the transfer
of TAGs from VLDL into HDL in exchange for CEs from HDL into
VLDL. These CE reach the liver inside of IDL or LDL.
15. Explain how cholesterol esters from HDL can reach the liver via
IDL and LDL
a. Once the TAGs in VLDL are cleaved by LPL they remnant is called
IDL. IDL gives apo C-II back to HDL and the remaining apo proteins
apo B-100 and apo E are recognized by LRP (LDL receptor related
protein) in the liver and taken up. The remaining IDL is converted to
LDL by Hepatic Lipase, the LDL gives apo E to HDL. Now it only has
apo B-100 which can be recognized by LDL receptors in the liver
and taken up.
16. Describe the delivery of cholesterol esters to the liver by HDL 2
via SRB-1 and using the phospholipase activity of hepatic lipase
a. Cholesteryl esters are taken up into the liver by scavenger receptors
SR-B1 and hepatic lipase (has both lipase activity acting on TAGs in
IDLs and phospholipase activity very unique). After binding to SR-B1
the phospholipid layer of HDL2 is opened by hepatic lipase and some
CEs enter the liver. The phospholipid monlayer closes again and
now the smaller HDL3 leaves the receptor. HDL3 can be changed to
HDL2 in the blood again.

15

17. Explain how oxidized LDL particles are formed and how they
are involved in foam cell development and atherosclerosis
a. oxLDL is formed from superoxides, nitric oxide, hydrogen peroxides
oxidizing phospholipids or apo B-100. They are more easily retained
by the ECM and proteoglycans and can be oxidized to ox-LDL. OxLDLs accumulate in macrophages via SR-A because they cannot be
recognized by LDL receptors. This leads to a formation of foam cells.
Plaque formation leads to atherosclerosis.
18. Discuss the significance of Lp(a) and LDL-B
a. oxLDL can be formed from LDL-pattern B which are small and dense
and penetrate more easily the endothelium.
b. Lp(A) is similar to LDL but has an additional apo(a) linked to apo B100 via a disulphide bond which shows unusual structure of
kringles. It is a structural analog to plasminogen. Apo(a) may
compete with plasminogen for the binding to fibrin and reduce the
removal of blood clots which could trigger MI or stroke.
19. Describe Type I, IIa, IIb, III, IV and V hyperlipidemias (causes,
lipid profile abnormalities, biochemical basis of clinical
manifestations) (On table)
20. Describe the biochemical basis for the use of statins and bile
acid sequestering agents in hypercholesterolemia
a. Statins inhibit as competitive inhibitors HMG-CoA reductase which is
the regulated enzyme of cholesterol synthesis. When less free
cholesterol is found in the liver then more LDL receptors are
synthesized. More LDL receptors can take up more LDL from blood
and reduce serum cholesterol
b. Bile acid sequesterants bind and trap bile acids/salts in the
intestines and lead to excretion via the feces. The enterohepatic
reuptake of primary and secondary bile acids is reduced and free
cholesterol in the liver will be used to synthesize new bile acids.
This reduces cytosolic free cholesterol levels in the liver and we find
more LDL-receptor synthesis.

16

Lecture: Hormonal regulation of fuel metabolism: The feed-fast


cycle
1. Explain metabolic homeostasis and describe the mechanisms
involved in the inter-tissue integration required for metabolic
homeostasis.
a. The hormones insulin and glucagon, epinephrine and cortisol are
the main hormonal regulators of blood glucose levels. CHO, lipid
and protein metabolism of tissues are afected. Insulin and glucagon
are hormones with opposite actions. Glucagon action is supported
by epinephrine, norepinephrine, cortisol and other insulin counter
hormones.
Mechanisms
required
are
Glycogenolysis,
gluconeogenesis, ketogenesis and lipolysis. Regulation by substrate,
serum level of hormones and nervous system.
2. Explain the special role of glucose in metabolic homeostasis
a. High levels of blood glucose after a meal lead in the liver to
glycolysis, glycogen synthesis, and also synthesis of fatty acids and
cholesterol. High levels of blood glucose also inhibits the release of
glucagon.
3. Describe in general the pathways leading to fatty acid synthesis
and cholesterol synthesis in the fed state starting with glucose.
a. Glucose is broken down into pyruvate by glycolysis. Through PDH an
acetyl-CoA can be used to initiate FA synthesis and Cholesterol
Synthesis. Both are stimulated by insulin.

17

4. Describe the roles of insulin and glucagon as the two major


hormones that regulate fuel storage and mobilization.

CHO
Metabolism

Insulin
(Dephosphoryla
tes via Protein
Phosphotases)
Glycogen
Synthesis and
Glycolysis. GLUT4 Mobilization,
PPP

Lipid
Metabolism

Synthesis of FA,
TAGs and
cholesterol in the
liver, VLDL, TAG
synthesis in fat
cells

Protein
Metabolism

Synthesis of
proteins in
muscle

Glucagon
(Phosphorylat
es via Protein
Kinase A)

Epinephrine

Glycogen
degradation and
Gluconeogenesi
s in Liver only

Glycogen
degradation in
liver and muscle

FA degradation
and usage of
FA degradation
ketone bodies in
and synthesis of
extra-hepatic
ketone bodies in
tissues. TAG
liver only
degradation in fat
cells
Degradation of
Usage of amino
proteins in
acids for
muscle mainly by
gluconeogenesis
cortisol

5. Describe the roles of epinephrine


and glucocorticoids in
regulation of fuel metabolism
a. Epinephrine in table above.
b. Glucocorticoids- Under stress situations, the pituitary gland releases
ACTH which stimulates release of cortisol from the adrenal cortex.
Cortisol leads in adrenal medulla to methylation of norepinephrine
to epinephrine and release of both catecholamines into the blood.
Epinephrine inhibits insulin release from -cells and stimulates
glucagon release from -cells of the pancreas. Cortisol induces PEP
carboxykinase and favors gluconeogenesis in the liver. Cortisol
leads to protein degradation in muscle. Cortisol leads to TAG
degradation in certain fat cells and release of fatty acids into the
blood. Epinephrine leads to glycogen degradation and TAG
degradation in fat cells.
6. Describe the effect of changes in the insulin/glucagon ratio and
plasma epinephrine levels on carbohydrate, lipid and protein
metabolism.
a. High insulin ratio (anabolic hormone) leads to glucose uptake (GLUT4). Glycogen synthesis, amino acid uptake, protein synthesis, usage
of branched-chain amino acids for energy metabolism. Low plasma
epinephrine levels.

18

b. Low Insulin/Glucagon ratio leads to protein degradation, amino acid


release (alanine-glucose cycle) uptake of ketone bodies and fatty
acids. This leads to a high plasma epinephrine ration because
cortisol is stimulating the release of the catecholamines from the
adrenal medulla.
7. Outline the metabolic changes that occur during the feed/fast
cycle.
8. Discuss pathways that are active/ inactive in each major
organ/tissue during the feed/fast cycle. Describe how these
pathways are controlled and coordinated in different metabolic
states. (Next Page)

19

20

Lecture: Introduction to nitrogen metabolism


1. Explain in broad terms the concept of the amino acid pool
a. The amino acid pool is defined as all the free amino acids in cells,
blood and extracellular fluid.
b. Write the sources of amino acids for the amino acid pool and
the utilization of amino acids from the pool
i. It is filled by dietary amino acids, filled by synthesized
nonessential amino acids or filled by amino acids
generated by protein degradation. Amino acids are taken
out of the pool for body protein synthesis, for synthesis of
specilalized products derived by conversion of amino acids,
and for degradation for energy metabolism in all cells or for
synthesis of glucose/ketone bodies in hepatocytes.
c. Define and enumerate the essential and nonessential amino
acids in humans
i. Essential- PVT TIM HALL: Phenylalanine, Valine, Tryptophan,
Threonine, Isoleucine, Mehthionine, Histidine, Arginine,
Leucine and Lysine.
1. Arginine is definitely dietary essential in children
2. **Deficiency of Phenylalanine and Methionine makes
Tyrosine and Cysteine essential (respectively).
ii. Non-essential- Everything else not on this list **
2. Distinguish between the two major mechanisms of intracellular
protein degradation:
a. Proteasome and ubiquitin- Tags proteins by binding of globular
non-enzymatic protein ubiquitin (ATP Dependent). Proteasome
recognized the ubiquitinated protein and cleaves the protein to
peptides (ATP dependent). These cleaved peptides enter Amino Acid
Pool. Degrades proteins synthesized by cells.
b. Lysosomal- Takes place in lysosomes with acid hyrdrolases.
Degradation is ATP-independent. Lysosomes degrade mainly
extracellular proteins, like plasma proteins or cell-surface
membrane proteins. Membrane prevents degradation of cytosolic
proteins allowing higher proton concentration inside lysosome.
3. Describe the role of liver and kidney in nitrogen metabolism and
excretion of non-protein nitrogenous substances
a. Liver- Gets its dietary amino acids from the portal vein. The liver
makes enzymes such as albumin and / Globulins which are
crucial. Amino acids not used by liver are released into the blood
amino acid pool, especially the dietary essential branched chain
amino acids (valine, isoleucine, leucine). Cells release nitrogen to
liver in the form of alanine or glutamine into the blood. Alanine
gives one nitrogen to the liver while glutamine gives two.
b. Kidney- Kidney excretes nitrogen mostly as Urea (From Liver). Urea
formed in the liver is transported to the kidneys where it is excreted
21

4.

5.

6.

7.

8.

in urine. Ammonia is also excreted glutamine ammonia


(glutaminase drives this reaction). Uric acid from metabolism of
purines. Creatinine from muscle cells.
Describe the mechanism of transport of amino acids in the renal
tubule and in GIT
a. Dietary amino acids are transported against a concentration
gradient by secondary active co-transport with sodium ions into the
intestinal mucosal cell. There are about 7 diferent transport
systems for amino acids which have overlapping specificities for
diferent amino acids. These transporters are also found in the
kidney.
Analyze and correlate clinical features and laboratory findings to
the biochemical basis of cystinuria and Hartnups disease
a. Cystinuria- Tubular reabsorption of Cystine is decreased. The
Cystine precipitates in the renal tubule. The COAL transporter is
deficient (Cystine, Ornithine, Arginine and Lysine).
b. Hartnups Disease- Defect in transport of neutral amino acids like
tryptophan. May lead to NAD+ (Niacin) deficiency (Pellagra).
Tryptophan can be used for synthesis of niacin. Low protein diet.
Represent the general scheme of amino acid catabolism
a. First step is the removal of nitrogen by transamination or
deamination which is followed by eventual degradation of the
carbon skeleton in the TCA cycle. Transamination transfers the
amino acid to an -ketoacid and generate another amino acid which
is glutamate. Transamination does not generate free ammonium
ions. Deamination generates free ammonium ions and it performed
in the liver and in the kidney where the ammonium ions are used for
the urea cycle or are released into urine respectively.
Distinguish the fates of C- skeletons of amino acids:
a. Glucogenic amino acids- leads to a degradation product that can
be used for gluconeogenesis. For example pyruvate or an additional
molecule in the TCA cycle
b. Ketogenic amino acids- Leads to acetoacetyl-CoA or acetyl-CoA.
These carbons cannot be used for gluconeogenesis but they can be
used for ketone body synthesis in the liver.
c. Glucogenic & Ketogenic amino acids- Tryptophan
d. Enumerate examples of amino acids in the three groups
a. Glucogenic- Glutamate, Glutamine, Proline, Histidine,
Arginine, Isoleucine, Valine, Methionine, Threonine, Tyrosine,
Phenylalanine, Asparagine, Aspartate, Alanine, Cysteine,
Glycine, Serine, Tryptophan
b. Ketogenic- Isoleucine, Phenylalanine, Tryptophan, Tyrosine
and purely Ketogenic are Leucine and Lysine
Revise the important TCA cycle intermediates formed from the
glucogenic amino acids and integrate their further fate in
metabolism (Draw this Out on left side of page)
22

a. Link [glutamate and glutamine] to the TCA cycle and


metabolism
b. Link [aspartate and asparagine] to the TCA cycle and
metabolism
c. Link [alanine] to pyruvate and metabolism

Lecture: Disorders of Amino acid catabolism


1. Phenylalanine and tyrosines
Phenylalanine

a. PKU I (Classic)- Phenylalanine levels in blood are elevated


resulting in mousey odor of urine. You have a decreased
pigmentation of skin and hair as tyrosine conversion to melanin is
inhibited by the elevated Phe levels (Inhibits tyrosinase). Deficienct
enzyme is Phenylalanine Hydroxylase (PAH). Need to make sure
infants are on a Phe free diet.
b. PKU II (Malignant)- Deficiency of dihydrobiopterin synthesis or
dihyrdrobiopterin reductase (BH2/BH4). Decreased neurotransmitter
synthesis. Treatment includes dietary Phe restriction and providing
dietary biopterin and precursors of the neurotransmitters. Worse
prognosis than PKU I. Deficient catecholamine formation, deficient

23

serotonin. BH4 is needed for L-Dopa and Serotonin. Sapropterin may


be used in some PKU patients (synthetic form of BH4

Tyrosinemias

a. Alkaptonuria- Inborn error of phenylalanine-tyrosine catabolism.


Deficiency in Homogentisic Acid Oxidase, and homogentisic acid
accumulates. Dietary restriction of Phe and Tyrosine may reduce
deposition of homogentisic acid. Manifestations is darkening of urine
on standing, discoloration of cartilage and CT.
b. Tyrosenemia Type I- Inborn error of phenylalanine-tyrosine
catabolism. Deficiency of fumaryl acetoacetate hydrolase. Cabbage
like odor of the urine. Tyrosine is needed for NT synthesis thats why
it is hard to exclude it from the diet.
2. Branched chain amino acids
Maple syrup urine disease

24

a. Dietary restriction of branched chain amino acids (leucine,


isoleucine, valine). Presents with poor feeding, vomiting, poor
weight gain. Ketosis and the characteristic odor of maple syrup in
the urine. Dietary supplementation with TPP (vitamin B1) may be
useful in those patients that have an enzyme with low coenzyme
affinity.
Methylmalonic aciduria

b. Methylmalonate levels in circulation were elevated. Causes


metabolic acidosis. Some children there is an improvement with
vitamin B12 Cobalamin. Deficient enzyme is Methylmalonyl CoA
mutase
3. Methionine and cysteine
Homocystinuria

25

a. Outline the two possible fates of homocysteine- Defect in the


metabolism of homocysteine. Deficiency of the cystathionine synthase. Homocysteine binds to connective tissue and disrupts its
structure. Some patients respond to oral vitamin B 6 (PLP).
Manifestations similar to Marfans. Homocysteine can also be
transferred to a serine to form cysteine. This carbon skeleton can for
succinyl-CoA for entry of the TCA cycle.
Lecture: Urea cycle
1. Identify the transport forms of ammonia from peripheral tissues
(alanine & glutamine). Indicate the role of glutamate
dehydrogenase
and
glutamine
synthetase
in
glutamine
formation. (P. 25 Cycle in top right Slide)

26

a. Glutamate Dehydrogenase can take free ammonia and Ketoglutarate and create Glutamate. Glutamine synthetase can take
glutamate and synthesize Glutamine.
2. Describe the transamination (aminotransferase) reaction and its
role in the transfer of amino groups. Indicate the importance of
vitamin B6 (pyridoxal phosphate) P. 25 Top Right

3. Describe the role of glutamate dehydrogenase in donating NH 3


for the urea cycle
a. Oxidative deamination by glutamate dehydrogenase forms free
ammonia. Glutamate -Ketoglutarate + free ammonia
4. List the reactions that form ammonia in the liver:
a. From Alanine: transamination requires amino acid transaminase and
PLP as a cofactor. The amino group is first transferred to Ketoglutarate to form glutamate.
b. From
Glutamate
oxidative
deamination
by
glutamate
dehydrogenase forms free ammonia.
c. From Glutamine by glutaminase recction free ammonia and
glutamate are formed.
d. Explain the formation of ammonia in the intestine and its
detoxification.
i. Bacterial ureases form ammonia in the colon. The
ammonia enters the portal circulation and is delivered to
the liver and a majority of the ammonia forms urea in the
liver.
e. Analyze the significance of intestinal ammonia formation in
patients with compromised liver function
i. Patients with Cirrhosis increases the amount of ammonia
entering systemic circulation because it is not being
converted to urea in the intestine. Ammonia is NOT being
detoxified.
5. Regarding the urea cycle,
a. Identify the sources of nitrogen for the formation of urea
i. The first N comes from Ammonia and the second N atom
from aspartate
1. Formation of aspartate from OAA
b. Recall the subcellular location of the urea cycle enzymes
and energy utilized for the formation of urea
27

i. Takes place in the Liver. Two reactions in the mitochondria


and three reactions in the cytosol. One urea molecule
needs 4 high energy bonds.
c. Explain the regulation of the urea cycle (regulatory enzyme
and mechanisms of regulation) and analyze the significance
of N-acetyl glutamate
i. N-acetyl glutamate (NAG) is an absolute activator of CPS I
(RLE)

d. Compare and contrast the five inherited disorders


associated with the urea cycle (Specify enzyme deficient,
correlate clinical and biochemical features, and generalize
the biochemical basis for management of hyperammonemia
for each of the disorder)
i. All the urea cycle disorders are characterized by increased
blood ammonia levels (Hyperammonemia) and blood
glutamine levels. Deficiency of CPS I or OTC (first two
enzymes) is most severe.
Disease Name

Enzyme
Deficient

Accumulating
Substrate

Treatment
Sometimes responds to
Arginine. Arginine forms
NAG and high levels
may stimulate CPS I

Hyperammone
mia Type I

CPS I

Ammonia

Hyperammone
mia Type II

OTC
X-Linked
and most
common

Carbomoyl
Phosphate/Orotic
Acid

Citrullinemia

ASS

Citrulline

Argininosuccini

ASL

Arginino

Arginine may enhance


citrulline excretion
Arginine may enhance
28

c Aciduria
Argininemia

Succinate
Arginase

Arginine

Argino Succinate
excretion
Diet of Essential Amino
acids excluding Arginine

e. Low protein/high carb diet, -Keto acid, prevention of stresses that


induce catabolic state. Long term: liver transplant.
f. Indicate the biochemical basis for the use of phenylacetate,
benzoic acid, -keto acids, arginine and carbamoyl
glutamate in the management of the urea cycle disorders
i. Phenylbutyrateundergoes
-oxidation
and
forms
phenylacetate. Phenylacetate combines with glutamine to
form phenylacetylglutamine and is excreted in urine.
ii. Benzoic Acid- combines with glycine to form hippuric acid
which can be excreted in the urine
6. Regarding acquired hyperammonemia:
a. Identify causes of hyperammonemia
i. Liver disease due to viral or drug indeced hepatitis,
alcoholic cirrhosis. In cirrhosis portal blood enters the
systemic circulation without going to the liver. Ammonia in
the intestine enters circulation causing neurotoxicity
b. Analyze
the
mechanisms
of
neurotoxicity
of
hyperammonemia
i. Elevated ammonia leads to -Ketoglutarate being
converted to glutamate via glutamate dehydrogenase
enzyme (Consumption of TCA cycle intermediates. This
leads to less ATP, reduced Na+/K+ activity. Osmotic
pressure and eventual neuronal cell death.
c. Explain the biochemical basis of use of antibiotics and
lactulose in the management of acquired hyperammonemia
i. Lactulose- is a disachharide that is resistant to digestion in
the small intestine. Normal flora digest in the colon to
produce lactic acid. Lactic acid is neutralized by NH 4+. More
Nitrogen excreted in feces.
ii. Antibiotics- reduction in bacterial urease in the gut.
Lecture: Conversion of amino acids to specialized products
1. Discuss the products of the amino acids listed below. For each
specialized product:
a. Specify the reactions and coenzymes involved
b. Analyze the biochemical significance of the specialized
product
2. Phenylalanine/ tyrosine:
a. Catecholamines (P. 32)
i.
Biosynthetic pathways of catecholamines
29

ii.

Explain the role of


degradation (P. 33)

MAO

and

COMT

during

iii.

Describe VMA and homovanillic acid as the


degradation product of the catecholamines & its
clinical significance
a. VMA levels may be measured to estimate
levels of epinephrine and norepinephrine
produced. High levels leads to headache,
sweating
and
tachycardia.
(Pheochromocytoma)
b.
b. Melanin- Albinism is where there is a complete or partial deficiency
of melanin in the skin, hair and eyes. Severe form of albinism
afects the eyes (Oculocutaneous Albinism). Deficiency of the
Tyrosinase enzyme so deficient conversion of tyrosine to melanin.
Increased risk of skin damage on exposure to sunlight and increased
risk of skin cancer. Melanin is a polymer of molecules.
3. Tryptophan:
a. Serotonin- Synthesized in the gut, platelets and CNS. Synthesized
from Tryptophan

30

i.

Explain the formation of 5-HIAA formation and


analyze its clinical significance
a. Serotonin is metabolized to 5-hydroxyindole acetic
acid by MAO. In carcinoid syndrome a tumor of
serotonin producing cells in GIT (APUD cells). This
causes an increase in 5-HIAA in urine. Symptoms
include diarrhea and bronchospasm.
b. Melatonin
i. Derived from serotonin and serotonin is derived from
tryptophan. Melatonin is a sleep inducing hormone.
4. Glutamic acid: (P 35)
a. GABA- inhibitory neurotransmitter in the central nervous system.

5. Histidine:
a. Histamine- Produced during allergic and inflammatory reactions by
mast cells. It is a vasodilator. Antihistamine drugs are used to
prevent the adverse efects of allergic reactions. They do not reduce
the formation of histamine they reduce the ability of histamine to
function as signal to other pathways (Receptor antagonist).
6. Arginine:
a. Nitric oxide- synthesized from arginine by nitric oxide synthase in
endothelium of blood vessels. Causes local vasodilation.
Nitroglycerin used in treatment of myocardial ischemia is converted
to NO that results in vasodilation of blood vessels and improvement
of blood flow to the heart. NO has short half life and degraded
rapidly

31

7. Creatine:
a. Amino acids required for formation- Arginine, Glycine and SAM
(S-Adenosyl Methionine)
b. Function of creatine-phosphate in the muscle- Accepts ~P
groups from ATP when the muscle is resting (temporary storage).
Creatine donates ~P groups to ADP when muscle is contracting.
c. Excretion product of creatine (P 36)
i.
Explain creatinine formation- Creatine Creatinine is
spontaneous

ii.

Interpret the clinical significance of serum


creatinine estimation- levels of ceratine kinase
(CPK/CK-MB) are a good indicator of Myocardial
ischemia/Myocardial damage. Serum creatinine levels are
a good indicator of renal function. Serum ceratinine levels
rise in acute or chronic renal failure.

8. Glutathione:
a. Amino acids required for formation- Glutamate, Cysteine and
Glycine
b. Revise the functions of glutathione- It is an intracellular
reducing agent (antioxidant). Important for the detoxification of
toxic hydrogen peroxide especially in RBCs. It is conjugated with
drugs to make them soluble, serves as a cofactor for enzymatic
reactions, and serves as an aid in rearrangement of protein disulfide
bonds.
9. Correlate each of the inherited/ acquired disorder listed below to
the biochemical basis of the manifestations and the associated
laboratory features
a. Parkinsons disease- Parkinsons is a loss of dopamine producing
cells
in
the
basal
ganglia.
Neurodegenerative
disorder.
32

Characterized by movement disorders: spasticity, tremors, loss of


memory, mood disturbances and postural instability. Symptoms are
improved by L-DOPA. L-DOPA is converted to dopamine in the brain
and that improves symptoms.
b. PheochromocytomaOverproduction
of
catecholamines.
Headache, sweating, tachycardia are predominant symptoms. This
can be cause by an adrenal medulla tumor, and patients will have
high urinary VMA.
c. Carcinoid syndrome- In carcinoid syndrome there is a tumor of
serotonin producing cells in GIT (APUD cells). This causes an
increase in 5-HIAA in urine (Serotonin is metabolized to 5-HIAA by
MAO). Symptoms include diarrhea and bronchospasm.
d. Albinism- Partial or complete deficiency of melanin in the skin, hair
and eyes. Deficiency of the tyrosinase enzyme which converts
tyrosine to melanin.
Lecture: Metabolic roles of folic acid and vitamin B12 (One carbon
metabolism)
1. Specify the one- carbon donors in metabolism and the groups
they donate (SAM, THF, Cobalamin)
a. SAM donates a methyl group
b. THF donates formyl, methenyl, methylene, methyl groups
c. Cobalamin donates methylcobalamin
2. Explain the formation of SAM and the reactions requiring SAM
a. SAM is required for methylation (CH3) reactions. It is formed from LMethionine by S-Adenosyl Methionine Synthetase. SAM donates a
methyl group in the synthesis of epinephrine, creatine,
phospholipids like PC. After it donates its methyl group a
homocysteine is formed.
3. Analyze the metabolism of homocysteine, vitamins/ coenzymes
required for metabolism & interpret the clinical significance of
homocysteine (Draw pic on Page 2 on left)
a. After SAM donates its methyl group it becomes homocysteine. It
can then be converted back to methionine (create SAM again) or
Cysteine.
b. To synthesize methionine from homocysteine the coenzymes
required are vitamin B12 and Methyl THF. To synthesize cysteine
from homocysteine you need PLP and Cystathione beta-synthase
c. Deficiency in either vitamin B12 or folic acid causes elevated plasma
homocysteine levels. This can lead to damage of blood vessels and
poses a risk for thrombosis. Megaloblastic anemia must be treated
with both folate and vitamin B12. Other cuases of B12 deficiency are
pure vegan diet, chronic pancreatitis, and terminal ileal disease
(Chrons disease). It can also lead to increased platelet aggregation,
lipid oxidation and calcification of atherosclerotic plaques.
33

4. Explain the formation of THF from folate and specify the


mechanism of action of their inhibitors
a. The precursor of THF is dietary folic acid and the required enzyme is
dihydrofolate reductase in a two step reaction and requires 2
NADPH.
b. Inhibitors- methotrexate and sulphonamides. Methotrexate inhibits
the human dihydrofolate reductase and sulphonamides inhibit the
bacterial dihydropteroate synthetase.
5. Summarize the formation of one carbon groups from amino acid
metabolism and the utilization of 1-C groups for nucleotide
synthesis
a. THF can accept 1-Carbon groups from amino acids and can donate
1-Carbon groups for biosynthetic reactions such as purine and
pyrimidine synthesis.
b. The amino acids that can donate carbon atoms to the 1-Carbon pool
are Glycine, Serine and Histadine. They all require THF.
6. Indicate the different forms of THF (formyl, methylene and
methyl) and reactions requiring the different forms of THF
a. When one carbon is donated to THF you can get formyl THF, methyl
THF, and methylene THF. Methyl THF is also the storage form of THF.
Degradation of histidine also needs THF. Histidine is degraded to
glutamate but in deficiency you will see increased levels of FLGlu in
urine after ingesting histidine. Histidinemia.
b. Methylene THF is used as a 1-C donor in the synthesis of purines
and thymidine
c. Methyl THF is used as a 1-C donor in the conversion of
homocysteine to methionine.
d. Formyl THF- (Couldnt find this)
7. Identify the reactions requiring B12
a. Methionine Synthase and methylmalonyl CoA mutase require B12
8. Compare and contrast the causes, clinical and biochemical
features of folate and vitamin B12 deficiency
a. Folate acid deficiency (develops in months) is caused by lacking in
fruit and vegetables in the diet. It can also be caused by drugs such
as methotrexate, trimethoprim (prevent purine and pyrimidine
biosynthesis kills blast cells and causes macrocytic anemia).
i. Rapidly growing cancers use folic acid, small bowel
malabsorption and sulfa drugs inhibits folic acid synthesis.
It is the most common vitamin deficiency in the US and
most common in pregnant women and alcoholics. Folic acid
before pregnancy reduces neural tube defects. Deficiency
results in homocysinuria and megaloblastic anaemia.
FIGIu accumulation in blood (histidinemia).
b. Vitamin B12 deficiency (develops in years) causes accumulation of
abnormal fatty acids (Cardiovascular disease). Pernicious anaemia
because inability to absorb B12 and can lead to megaloblastic
34

anaemia. Causes may be a pure vegan diet, chronic pancreatitis


and terminal ileal disease. Increased methylmalonate in
circulation.
9. Indicate the role of intrinsic factor in vitamin B12 absorption
a. Inability to absorb B12 because the parietal cells are injured. So B12
cant be absorbed.
10. Justify the mechanism of the occurrence of folate trap in B 12
deficiency
a. When you have a B12 deficiency you have a folate deficiency. That is
because B12 and methylfolate is required for the homocysteine to
methionine reaction however with B12 deficiency all folate gets
trapped as methyl THF.
Lecture: Purine metabolism
1. Identify the differences between N-base, nucleoside and
nucleotide with examples
a. Addition of a pentose sugar to the nitrogenous base produces a
nucleoside. Addition of either 1, 2, or 3 phosphate groups to a
nucleoside
produces
a
nucleoside
mono/di/triphosphate.
Nucleosides
are
like
Ribose,
deoxyribose,
cytidine
and
deoxyadenosine. Nucleotides with be Purines/Pyrimadines.
2. Regarding purine biosynthesis
a. Enumerate the C & N donors of the purine ring (amino acids
and 1-C groups) (P 13)

b. Outline purine biosynthesis (Draw left P 14 top left)


i.
Ribose-5-phosphate comes from the HMP pathway. All
enzymes necessary for de novo purine synthesis are found in
the cytoplasm of the cell. Before synthesis can take place
PRPP is produced from the ribose-5-phosphate and ATP by
ribose phosphate pyrophosphokinase. The RLS of purine
synthesis is PRPP 5-phosphoribosylamine and this reaction
is catalysed by glutamine PRPP amidotransferase. The rate of
this reaction is controlled by high concentrations of glutamine
and PRPP. This reaction is also inhibited by the purine
nucleotides, AMP, GMP, and IMP. This pathway produces an
35

IMP which can be used to form adenosine (requires GTP) or


guanine (requires ATP). The process of creating an IMP
molecule requires 4 ATP total and 2 molecules of THF.
ii.
The reaction of IMP AMP requires GTP and
adenylosuccinate synthetase. The reaction of IMP GMP
requires
ATP
and
GMP
synthetase.
These
two
Monophosphates are converted to NDP and NTP by
nucleoside
monophosphate
kinases
and
nucleoside
diphosphate kinases respectively.
c. Discuss the regulation of purine biosynthesis (Identify the
regulatory enzyme and positive and negative modulators)
(Above)
d. Correlate the mechanism of action of sulfa drugs,
trimethoprim, methotrexate, mycophenolic acid to their
clinical application
i.
Sulfonamides competitively inhibit bacterial synthesis of folic
acid. Because purine synthesis requires THF (2 molecules)
the sulfa drugs slow down this pathway in bacteria.
Sulfonamides are PABA analogues that inhibit the synthesis
of folate in bacteria.
ii.
Methotrexate is a structural analog of folic acid and inhibits
the reduction of dihydrofolate to THF catalysed by
dihydrofolate reductase. These drugs limit the amount of THF
available for use in purine synthesis and slow down DNA
replication in mammalian cells. They are useful in treating
rapidly growing cancers (leukemia).
iii.
Mycophenolic acid is an inhibitor of inosine monophosphate
dehydrogenase. It is a reversible uncompetitive inhibitor of
IMP Dehydrogenase. Deprives rapidly proliferating T and B
cells of key components of nucleic acids. Drug is used to
prevent graft rejection. This doesnt let you synthesize GMP
e. Explain the formation of deoxyribonucleotides from
ribonucleotides (Specify the coenzyme and inhibitor)
i.
Reduction is catalysed by ribonucleotide diphosphate
reductase. Two important inhibitors are dATP and
hydroxyurea (this is why ADA deficiency is bad dATP inhibits
ribonucleotide reductase). The coenzyme is Thioredoxin.
3. Regarding salvage pathway of purine bases
a. Identify the reactions catalyzed by HGPRT and APRT
i. HGPRT catalyse the reactions of Hypoxanthine IMP and
Guanine GMP.
ii. APRT catalyzes the reaction of Adenine AMP
b. Correlate the clinical and biochemical features of Lesch
Nyhan syndrome (Specify enzyme deficient)
i. It is an X-linked genetic disorder that results in a deficiency of
HGPRT. Results in an inability to salvage purine hypoxanthine
36

and guanine. The end product degradation of hypoxanthine


and guanine is uric acid. Therefore children with lesch-nyhan
have excess uric acid in urine. Orange crystals are found in
babies diapers. This disease causes severe mental
retardation, self-mutilation (biting lips), and involuntary
movement, gout. You find increased PRPP levels and an
increase in de novo purine synthesis (this is why you have
increased uric acid).
4. Regarding purine nucleotide catabolism
a. Explain uric acid formation
i. Purines arent essential components of the diet so purines
from the diet are degraded to uric acid rather than
salvaged.
b. Analyze causes of hyperuricemia, correlate the biochemical
basis of the clinical features and explain the biochemical
basis of use of allopurinol in the management of
hyperuricemia
i. Deficiency in GALT (metabolize galactose), Glucose 6phosphotase, and Aldolase B results in hyperuricemia. Gout
is characterized by hyperuricemia; acute arthritic joint
inflammation caused by deposition of uric acid crystals.
Allopurinol is a non-competitive inhibitor of xanthine
oxidase. Causes excretion of hypoxanthine and xanthine
instead of urate. Better water solubility so excreted from
kidney. No buildup of uric acid.
5. Propose the biochemical basis of SCID (ADA deficiency)
a. Adenosine deaminase deficiency causes SCIDS. Guanosine and
inosine are better substrates for purine nucleoside phosphorylase
than is adenosine. DNA is not synthesized in T-cells and B-cells
because of an accumulation of dATP. Extremely large buildups of
dATP in red cells.
Lecture: Pyrimidine metabolism
1. Regarding pyrimidine biosynthesis
a.
Enumerate the donors of C & N atoms to pyrimidine ring
(P 22)

37

b.

c.
d.

e.

f.

Outline pyrimidine biosynthesis (UTP & CTP). Distinguish


purine and pyrimidine biosynthetic pathways
i. Pyrimidines are synthesized before attachment to the
ribose-5-phosphate which is donated by PRPP. The rate
limiting step and the committed step for pyrimidine
synthesis is the synthesis of carbamoyl phosphate from
glutamine and CO2 by carbamoyl phosphate synthetase II
(CPS II). CPS II is regulated by its products UDP and UTP
inhibit its activity. Folic acid derivatives are NOT used in
pyrimidine synthesis. This pathway creates UMP which can
be converted into CTP or dTMP. Utp is converted to CTP by
CTP synthase. Glutamine is the nitrogen donor.
Summarize the regulation of pyrimidine biosynthesis
(regulatory enzymes, regulators) above
Explain the synthesis of dTMP
i. Thymidylate synthase uses THF to produce dTMP from
dUMP. This reaction is indirectly inhibited by Methotrexate.
The reason it is indirect is because this reaction requires
THF, methotrexate slows down the synthesis of THF.
Indicate the mechanism of action of 5 fluoro uracil and
methotrexate and their clinical application
i. 5-fluorouracil is a uracil analog. The drug is given with
thymidine to boost efectiveness. It is converted to dFUMP
which inhibites thymidylate synthetase. In cancer cells 5flurouracil is incorporated into the RNA. This RNA with 5flurouracil is more detrimental to cancer cells than to
normal cells.
ii. Methotrexate is a structural analog of folic acid and inhibits
the reduction of dihydrofolate to THF catalysed by
dihydrofolate reductase. These drugs limit the amount of
THF available for use in purine synthesis and slow down
DNA replication in mammalian cells. They are useful in
treating rapidly growing cancers (leukemia).
Regarding orotic aciduria
i. Specify enzyme deficiency and correlate biochemical
features to the clinical features
1. A deficiency in Orotate phosphoribosyltransferase
(Orotate build up) and OMP decarboxylase (Orotidine
5-monophosphate) causes orotic aciduria. Causes
abnormal growth, megaloblastic anemia, and
excretion of large amounts of orotic acid in the urine.
ii. Compare and contrast orotic aciduria due to defect
in pyrimidine biosynthesis and defect in urea cycle
1. An acquired form of the disease may appear in
patients being treated for cancer with a pyrimidine
38

analog. A deficiency in these enzymes doesnt let


you create UMP which leads to a build up of their
respective precursors (Oratate Orotic Acid). A
deficiency in OTC (2nd enzyme) in the urea cycle also
leads to orotic aciduria.
2. Indicate the end products of catabolism of the pyrimidine
nucleotides
a.
Not sure about this one. Unlike purines, pyrimidine rings can be
opened and degraded to highly soluble structures that can serve as
precursors for other biomolecules. Pyrimidines can also be salvaged
by pyrimidine phosphoribosyltransferase where PRPP is the source
of the ribose phosphate.

Lecture:

Serum proteins and associated disorders

1. Describe
how
plasma
proteins
can
be
separated
by
electrophoresis
and
classify
plasma
proteins
based
on
electrophoretic mobility.
a. Serum is preferred for analysis of proteins. Serum proteins can be
separated by charge using electrophoresis. The most negatively
charged plasma protein and that travels the most is albumin,
followed by -1, -2, beta and then gamma proteins.
2. Describe the functions of serum albumin and globulins.
a. Albumin- most abundant serum protein and contributes to the
osmotic pressure of the plasma. It cannot cross the capillary wall
into the interstitial space and it helps to retain water in the
intravascular space. When serum albumin levels are low there is an
increased amount of water in the interstitial space leading to
edema. Another efect is low colloid osmotic pressure. Individuals
with congenital analbuminemia appear normal. Albumin can be
glycosylated at high serum glucose levels (could use for a diabetes
text). Presence of albumin in urine implies damage to basement
membrane of the glomerulus. Albumin since it is so negative can
bind Calcium. An increase in plasma pH results in higher binding of
39

calcium to albumin resulting in hypocalcemia. Albumin also


transports free fatty acids in its hydrophobic pockets. It can also
bind hormones like thyroxine and serve as a reservoir for free
hormone. Albumin binds bilirubin as well and transports it to the
liver for conjugation, however drugs (aspirin) can displace bilirubin
from albumin and lead to kemicterus in infants (bilirubin
accumulation in the brain).
b. Globulins- Transport, enzymes and inhibition of proteases (more
specific below Q. 5)
3. Indicate the role of proteins of the complement system (Couldnt
find in notes)
4. Predict three common causes of hypoalbuminemia (liver disease,
nephrotic
syndrome, protein malnutrition) and explain the
biochemical
basis
for
the
occurrence
of
edema
in
hypoalbuminemia
a. Decrease synthesis of albumin is a result of a low protein diet
(malnutrition) or chronic liver disease (cirrhosis). Increased loss of
albumin can be a result of nephrotic syndrome where albumin is lost
via the urine.
5. Distinguish the functions of proteins that are found in the
a. 1-globulin fraction (1- antitrypsin, fetoprotein, retinol
binding protein, transcortin)
1. 1-antitrypsin- an inhibitory protein against neutrophil
elastase in alveoli. A deficiency of 1-antitrypsin permits
neutrophil elastase to destroy lung. May lead to
emphysema. This protein is normally synthesized in the liver
and is N-glycosylated. M allele is the normal allele and Z/S
allele are defective alleles. Individuals who are homozygous
for the Z allele (Pi ZZ) have high risk of developing
pulmonary and liver disease. Smoking will increase the risk
for emphysema because smoke oxidizes methionine in 1antitrypsin which is needed to bind elastase.
2. fetoprotein (AFP)- is abundant in fetal plasma and may
have similar function to albumin. High maternal serum AFP
indicates fetal neural tube defects. Low maternal serum AFP
is an indicator of Down syndrome. AFP in adults is used as a
marker for hepatocellular cancers.
3. Transcortin- synthesized in the liver and in blood and is the
main transport protein for cortisol
4. Retinol binding protein (RBP)- transports retinol in blood.
Delivers retinol from the liver stores to the peripheral
tissues.
b. 2-globulin
fraction
(2
macroglobulin,
haptoglobin,
ceruloplasmin)
1. 2 macroglobulin is one of the largest serum proteins and
acts as protease inhibitor. It binds to and inactivates
40

proteases like plasmin and thrombin. Levels of 2


macroglobulin are elevated in nephrotic syndrome. Loss to
urine is prevented by its large size
2. Haptoglobin- A serum protein that binds to free hemoglobin
in circulation. Haptoglobin-Hemoglobin complex cannot be
excreted by the kidney, thus preventing loss of hemoglobin
(iron and globin). Low serum free haptoglobin levels are
found in patients with acute hemolysis (haptoglobinhemoglobin complexes increase) and can be used to monitor
patients with hemolytic anemia.
3. Ceruloplasmin- A copper containing plasma protein and is
synthesized and secreted by the liver. Ceruloplasmin has
ferroxidase activity and helps in the oxidation of Fe2+ to Fe3+
so it can be encorporated into transferrin. Low ceruloplasmin
levels are found in wilsons disease. Due to less attachment
of copper to ceruloplasmin, copper accumulates in tissues
Kayser-Fleischer rings.
c. -globulin fraction (transferrin, hemopexin, lipoproteins)
1. Transferrin is a transport protein for iron which needs to be
in the ferric form. Trasnsferrin transports iron between
intestine, liver, bone marrow and spleen. Each transferrin
can bind 2 Fe3+. Patients with iron deficiency have low
transferrin saturation.
2. Hemopexin- Binds to free heme in circulation and prevents
the loss of iron by the kidneys.
3. LDL/-Lipoproteins- Most negatively charged because of
apo-b-100. Transport cholesterol.
d. -globulin fraction (Immunoglobulins: Ig G, M, A, D, E)
1. Are immunoglobulins produced by activated B lymphocytes
(plasma cells)
2. IgM- first antibody to be produced in response to an antigen
3. IgG- antibody produced on repeated exposure to same
antigen. It can also cross the placenta and confer immunity
to the fetus and newborn
4. IgE- secreted in response to an allergen
6. Explain the consequences of inherited 1-antitrypsin deficiency on
the liver and the lungs and discuss the effects of smoking on the
structure of 1-antitrypsin.
a. Individuals who are homozygous for the Z allele (Pi ZZ) have high
risk of developing pulmonary and liver disease (accumulates in
liver). Smoking will increase the risk for emphysema because smoke
oxidizes methionine in 1-antitrypsin which is needed to bind
elastase.
7. Interpret the utility of C-reactive protein as an inflammatory
marker

41

a. Acute phase proteins are increased in any inflammatory disorder.


Cytokines released during inflammation stimulate hepatic synthesis
of these proteins. Typical examples include C-reactive protein,
ceruloplasmin, and haptoglobin. CRP levels are measured by
specific tests to monitor the progress of an inflammatory reaction.
8. Recognize the serum protein electrophoretic pattern in the
following diseases (hypoalbuminemia, 1-antitrypsin deficiency,
polyclonal gammopathy, monoclonal gammopathy) Draw out all
the charts and say what is elevated on page before blank
Lecture: Heme synthesis and porphyrias (Draw last page out with
Lippincott additions)
1. Describe in detail ALA synthase and ALA dehydratase reactions.
a. ALA Synthase- It has two isozymes. ALAS1 for the liver and ALAS2
for the RBC. They are regulated in diferent ways. ALA synthase
uses a succinyl-CoA and glycine to create ALA. ALA synthase needs
PLP as a coenzyme as it decarboxylates Glycine.
i. ALAS1- Drugs increase ALAS1 activity as they lead to CYP 450
synthesis which needs heme. Low intracellular heme
concentration leads to ALAS1 synthesis. Heme synthesis stops
when heme is not incorporated into proteins and as result,
heme and hemin accumulate. Hemin decreases the synthesis
of ALAS1 and also the mitochondrial import of the enzyme
from cytosol. Down-regulated by heme/hemin acumulation
ii. ALA Dehydratase (Porphobilinogen Synthase)- Accumulation of
heme will lead to MORE globin chain synthesis. Synthesis of
heme in eythroid cells is under control of erythropoietin and
availability of intracellular iron. ALAS2 is found in fetal liver
and adult bone marrow and a deficiency leads to X-linked
sideroblastic anemia. Down-regulated at low amounts of
intracellular iron.
2. Outline heme synthesis starting from glycine and succinyl CoA to
the formation of heme
a. Refer to drawing on left
3. Distinguish between the different regulatory processes of heme
synthesis in the liver and in the erythroid cells. (Question 1)
4. Explain how pyridoxine deficiency affects heme synthesis.
a. PLP is needed as a coenzyme for ALA synthase and heme synthesis
cannot start if this is deficient.
5. Discuss the effects of lead poisoning on heme synthesis.
a. This metal can interact with the zinc cofactors for ALA dehydratase
and ferrochelatase. ALA and protoporphyrin IX accumulate in urine.
Ferrochelatase
needs
zinc
as
well
as
ALA
hydratase
(porphobilinogen synthase). Leads to accumulation of ALA and
protoporphyrin IX.
42

6. Categorise the different types of porphyrias based on clinical


manifestations such as abdominal pain, photosensitivity and
neuropsychiatric symptoms.
a. Acute Intermittent Porphyria- Deficiency of HMB Synthase. ALA and
porphobilinogen in blood and urine. ALA activity is high because
ALA synthase is not feed-back inhibited by heme or hemin (none is
being created) so the synthesis of the ALA synthase enzyme is
increased. AIP can be diagnosed by urine color because on exposure
to light and air it changes from the colorless porphobilinogen to
dark colored urine (porphobilin). Symptoms include very severe
abdominal pain, high agitate state, mental disturbance, weakness of
lower extremeties. Patients are NOT photosenstivive as HMB cannot
be formed. ALA and porphobilinogen can act as neurotransmitters.
You do not administer barbiturates to these patients because it
stimulates the synthesis of CYP450 and that can worsen the
situation.
b. Congential Erythropoietic Porphyria (Does not efect liver)Deficiency of Uroprophyrinogen III Synthase. Autosomal
recessive trait that is characterized by extreme photosensitivity
and reddish brown teeth with werewolf features. Hemin
infusion would not help because erythroid cells contain ALAS2 which
is not inhibited by hemin rather it is stimulated by high intracellular
concentration of iron. You see Uroporphyrin I and Coproporphyrin I
in the urine.
c. Porphyria
Cutanea
Tardacaused
by
a
deficiency
in
uropophyrinogen decarboxylase. Uroporphyrin accumulates in
the urine and it is the most common porphyria and patients are
photosensitive. Type I is sporadic and Type II is familial.
Accumulation of uroporphyrin in the skin and also the liver. Erosions
and bullous lesions in sun-exposed areas of the patients are
produced by minor trauma because of increased skin fragility. Urine
is red upon release due to high levels of red uroporphyrin III.
7. Describe the biochemical basis for the clinical features (abdominal
pain and photosensitivity, dark colored urine) occurring in acute
intermittent porphyria, congenital erythropoitic porphyria and
porphyria cutanea tarda. Indicate the deficient enzyme and
products accumulating in the above mentioned disorders. (read
above its all there)
Lecture: Heme degradation and jaundice
1. Outline the steps in the degradation of heme to bilirubin in the
macrophages.
a. RBC half a lifespan of about 120 days. Old RBCs are sequestered by
the spleen. RBCs contain hemoglobin that is broken down to heme
and globin. Heme Oxygenase converts heme to biliverdin (green
43

pigment). This reaction also leads to the porphyrin ring being


cleaved, only reaction in body that creates Carbon monoxide and
converted the ferrous iron to ferric (3+).
b. Next the biliverdin is acted upon by bilverdin reductase which forms
bilirubin (orange/yellow color). This is known as unconjugated
bilirubin or free bilirubin.
2. Explore the relationship of serum albumin and the transport of
bilirubin
a. Bilirubin formed in macrophages is not water soluble. It binds in
blood to albumin for transport. This bilirubin is known as
unconjugated bilirubin. Binding of bilirubin to albumin prevents it
from being excreted in urine. Many drugs can displace bilirubin from
albumin and this can lead to a lot of free bilirubin that can cross
blood brain barrier causing kenicterus in children. This unconjugated
bilirubin is the taken up by the liver by specific transporters and is
bound to the intracellular liver protein ligandin.
3. Explain the biochemical consequence of a block in each of the
steps in the uptake and conjugation of bilirubin in liver.
Distinguish the activity of UDP-glucuronyl transferase in adults
versus premature babies.
a. Bilirubin is converted to conjugated bilirubin by the addition of two
molecules of glucuronic acid. The enzyme is microsomal UDP
glucuronyl transferase creating conjugated bilirubin. If you block the
uptake of bilirubin into the liver you will have high circulating levels
of unconjugated bilirubin. If you block conjugation of bilirubin you
will have kernicterus in children (specific defects discussed later).
b. Newborn infants have low activity of hepatic UDPglucuronyl
transferase (premature have even lower activity). You have jaundice
by 2nd or 3rd day of life that usually clears by the 7 th day of life.
Increased destruction of RBCs after birth overloads the livers
capacity to conjugate bilirubin.
4. Analyze the steps in the processing of conjugated bilirubin in the
intestine and its excretion in feces and urine
a. After bilirubin has been conjugated it is actively transported into the
bile canaliculus against a concentration gradient by a specific ABC
transporter. Conjugated bilirubin is a component of bile and is
released into the second part of the duodenum via the common bile
duct. Conjugated bilirubin is acted upon by bacterial flora in the
large intestine. Conjugated bilirubin undergoes deconjugation and is
converted to urobilinogen (colorless). Bacterial action on
urobilinogen forms stercobilin (brown) that is lost in the feces and
gives the feces its brown color. Some urobilinogen is absorbed from
the gut into portal blood. A majority of urobilinogen is lost in the
urine as urobilin (light yellow color).

44

5. Identify the sequential steps in heme catabolism beginning with


heme till the formation of urobilinogen and stercobilin; identify
the various tissues/organs involved in each step
a. (Refer to drawing on left as well). Heme is converted to biliverdin by
Heme oxygenase in the macrophage. The biliverdin is then
converted to bilirubin by bilverdin reductase once again in the
macrophage. (This process happens in reticuloendothelial system)
b. The free bilirubin is the transported in the blood by albumin which
transports it to the liver for conjugation. At the liver the bilirubin is
taken up by specific receptors and is bound intracellularly to a
protein called ligandin.
c. Within the liver UDP glucoronyl transferase adds 2 mols of
glucuronic acid via UDP-glucuronic acid. The conjugated bilirubin is
the actively transported into the bile canaliculus against a
concentration gradient by a specific ABC transporter.
d. Conjugated bilirubin is a component of bile and is released into the
second part of the duodenum via the bile duct. Conjugated bilirubin
is acted upon by bacterial flora converted into the colorless
urobilinogen. Urobilinogen can be converted to stercobilin giving
feces the brown color. It can also be reabsorbed by the portal
system and sent back to the liver. However a majority is lost in urine
as urobilin (light yellow).
6. Compare and contrast conjugated and unconjugated bilirubin with
reference to chemical composition, water solubility, tissue
deposition, excretion in urine and common clinical conditions
where they are elevated
a. Conjugated bilirubin is water soluble. You have high levels of
unconjugated bilirubin in kernicterus, prehepatic jaundice, and
hepatic jaundice. You have elevated levels of conjugated bilirubin in
hepatic jaundice and post hepatic jaundice. Free bilirubin is not
excreted in urine because it is bound to albumin. Conjugated
bilirubin can be converted to urobilinogen which can be converted
to urobilin (urine) or stercobilin (feces).
7. Distinguish conjugated and unconjugated hyperbilirubinemia
theoretically using lab tests
a. Van den Bergh reaction- Bilirubin reacts with diazo reagent to form a
red colored complex. Conjugated bilirubin reacts rapidly with the
reagent and is called direct bilirubin.
b. Total bilirubin- the reaction is also carried out in the presence of
methanol because unconjugated bilirubin is water insoluble and
reacts in the presence of methanol. (Indirect reacting)
c. Total bilirubin Direct (conjugated) bilirubin = indirect
(unconjugated) Bilirubin.
d. Hyperbilirubinemia is characterized by serum bilirubin > 2 mg/dL
8. Explain the biochemical mechanisms involved in the development
of physiological jaundice of the newborn
45

a. Newborns have a low activity of hepatic UDP glucuronyl transferase.


This leads to less conjugated bilirubin and more free bilirubin in the
serum. This is caused by an increased destruction of RBCs after
birth which overloads the livers capacity to conjugate bilirubin (high
unconjugated bilirubin levels in newborn). Unconjugated bilirubin
can now corss the blood brain barrier and deposit in the basal
ganglia of the brain resulting in kernicterus as it is lipid soluble.
(kernicterus can also be caused by drugs such as salicylates,
sulfonamides that compete with bilirubin for albumin binding
increasing the amount of unconjugated bilirubin in the blood). This
disorder is characterized by severe neurological symptoms.
9. Explain the rationale of phototherapy and phenobarbital in the
therapy for premature babies with jaundice.
a. Phototherapy in neonatal jaundice in premature babies with
jaundice is helpful because light converts bilirubin to more polar,
water soluble isomers, that can be excreted in bile without
conjugation.
10.
Distinguish prehepatic, hepatic and post hepatic jaundice
based on lab data.
Serum
Serum
Serum
Urine
total
conjugate unconjugat
Urine
Condition
Urobilinog
Bilirubi
d
ed
Bilirubin
en
n
Bilirubin
Bilirubin
Prehepati

Absent

c Jaundice
Hepatic

Present
N or or
Jaundice
Posthepat
ic
Jaundice

N
Present
/Absent
(Obstructi
ve)
11. Interpret laboratory data in the different types of jaundice
(Look at table with drawings)
a. Prehepatic: sickle cell anemia, G6PD deficiency (Write
disorders by respective drawings)
b. Hepatic: alcoholic cirrhosis, hepatitis
c. Obstructive jaundice due to gall stones or cancer of head of
pancreas
12. Differentiate between inherited disorders: Gilberts syndrome
and Crigler-Najjar syndromes I and II based on the pathogenetic
mechanisms and biochemical alterations
a. Crigler Najjar Type I- most severe and is associated with an almost
complete deficiency of the enzyme bilirubin glucuronyl transferase.

46

Serum bilirubin levels acan reach upto 50 mg/dL and result in


jernicterus and mental retardation. Management of crigler-najjar
syndrome type I is daily phototherapy.
b. Crigler Najjar Type II- (Arias syndrome) has a lower activity of
bilirubin glucuronyl transferase (10-20% of normal). Characterized
by jaundice but less severe than type I. Children respond to
phenobarbital (induces the enzyme). Regular phototerapy is also
used.
c. Gilberts syndrome- present in 3-7% of the population and is the
most common. Characterized by mild jaundice following stress or
starvation. UDP glucuronyl transferase activity is about 50% of
normal. Mild increase in unconjugated bilirubin.
d. Dubin-Johnson syndrome- inherited deficiency of the ABC
transporter that transports conjugated bilirubin from the hepatocyte
into the biliary canaliculus. Characterized by elevated levels of
conjugated (direct) bilirubin in circulation.
Lecture: Hemostasis
1. Outline the four phases of hemostasis.
a. Vascular Spasm/Vasoconstriction
b. Platelet plug formation/primary hemostasis
c. Blood coagulation/secondary hemostasis
d. Dissolution of the fibrin clot/tertiary hemostasis
2. Discuss vascular spasm and the role of endothelin.
a. Trauma to the vessel wall results in smooth muscle contraction.
Contraction is caused by local myogenic spasm, factors released
from the injured vessel wall (endothelin and serotonin) and nervous
reflexes. This cannot cause long term cessation of bleeding.
3. Describe the platelet plug formation.
a. Greatly limits the loss of blood from the circulation by forming plugs.
Small cuts in the blood vessels are often sealed by platelet plugs.
Platelets first adhere to the damaged surface and activate. Then
you have platelet aggregation.
b. Platelet adhesion and activation is facilitated by endothelial injury.
Platelet adhesion is mediated by platelet receptors glycoprotein
Ib/Ia. These receptors bind to ligands that are components of the
subendothelial matrix (collagen, von Willebrand Factor)
c. Platelet GpIa binds to collagen. Leads to development of
pseydopods to promote platelet-platelet interactions. vWF binds to
the platelet receptor glycoprotein Ib resulting in changes in the
platelet membrane. This binding also exposes GPIIb/IIa for binding
of fibrinogen.
d. vWF acts as a bridge between specific glycoproteins (GP Ib) on the
surface of platelets and collagen fibers. It facilitates platelet

47

4.
5.

6.
7.

adhesion to the vessel wall and platelet aggregation. Complexes


with factor VIII; carries it, stabilizes it and prevents its degradation.
vWF deficiency is associated with a defect in the formation of the
platelet plug (primary hemostasis)
Explain the role of integrins GPIb and GPIa. (Question above)
Describe the role of platelet-activating factors (ADP, serotonin,
TXA2 and thrombin).
a. ADP- released from the platelet granules. It is a potent platelet
activator, it promotes platelet-platelet contact, and platelet
adherence. Binding of ADP to its receptor facilitates the release of
Ca2+ and decerases cAMP levels (both of which facilitate platelet
aggregation). Binds to receptor on platelet membrane leading to
further unmasking of GpIIb/GpIIIa binding sites which bind
fibrinogen.
b. Once ADP binding releases Ca2+ phospholipase A2 is stimulated and
it creates TXA2 from arachidonic acid. This leads to platelet
aggregation by activating other platelets. Difuses out of the cell to
enhance vasoconstriction.
c. Serotonin- function sin the same way as endothelin. Contracts the
vessel in response to trauma. Part of the vascular spasm phase of
hemostasis.
d. Conversion of fibrinogen to fibrin requires thrombin. Fibrinogen is a
plasma protein that is synthesized by the liver and cleaves the
fibrinopeptides of fibrinogen to form the fibrin monomer. The fibrin
monomers aggregate and are linked to each other via hydrogen
bonds, forming the fibrin polymer (soft clot). Formation of a hard
clot is mediated by factor XIII activated by thrombin.
Describe the role of integrin GPIIb/IIIa in platelet aggregation.
(Above)
Describe the coagulation cascade, including the extrinsic,
intrinsic and common pathways.
a. Aim is to convert soluble fibrinogen to insoluble fibrin (threads that
stabilize the platelet plug) Requires thrombin.
b. Extrinsic (draw these out)
i.
Tissue Injury leads to release of tissue factor (factor III or
thromboplastin).
ii.
Factor VII is activated by tissue factor to VIIa.
iii.
Factor VIIa and tissue factor III in the presence of Ca 2+ and
platelet phospholipids activate factor X
iv.
Xa (active factor X)
c. Intrinsic
i.
Rough endothelial surface exposure of collagen
activation of factor XII
ii.
XIIa acts on XI to activate it
iii.
Xia acts on IX to activate it
iv.
Thrombin activates factor VII VIIa
48

IXa, VIIIa, platelet phospholipids and Ca 2+ activate factor X to


Xa
d. Common Pathway
i.
Thrombin acts on factor V Va
ii.
Xa combines with Va, tissue phospholipids and Ca 2+ to form
prothrombinase complex.
iii.
Prothrombinase complex splits prothrombin to form thrombin
iv.
Thrombin acts on fibrinogen to fibrin
v.
Thrombin activates factor XIII to XIIIa
vi.
Fibrin monomers are covalently crosslinked by factor XIIIa to
form cross-linked fibrin
e. Thrombin production from prothrombin needs factor Xa. Ultimate
aim of hemostasis is conversion of fibrinogen to fibrin and
stabilization of fibrin which requires thrombin IIa
8. Indicate the role of calcium in hemostasis (Above)
9. Discuss the role of vitamin K in the post-translational
modification of clotting factors
a. Vitamin K is required for the hepatic synthesis of prothrombin
(factor II), VII, IX, X, protein C and S. Vitamin K dependent decarboxylation of glutamic acid residues of the above proteins.
This process forms mature clotting factors and is capable of
activation. This carboxylation allows Ca 2+ binding because of two
adjacent negatively charged carboxylate groups. Clotting factor Ca2+
complex can then bind to phospholipids on the platelet membrane.
10. Predict the clinical manifestations in a patient with vitamin K
deficiency
a. The extrinsic and intrinsic pathways will not work because the
factors wont be synthesized.
11. Discuss the regulation of coagulation, and fibrinolysis; outline
the role of PGI2, antithrombin III, protein C, TFPI, t-PA and
plasmin.
a. PGI2- prevents platelet aggregation. Increases cAMP levels within
platelets and inhibits platelet activation. It is a thromboxane
antagonist.
Coagulation
automatically
initiates
fibrinolysis.
Hemostasis controller.
b. Antithrombin III- binds to and inhibits factor Xa and thrombin (IIa).
Heparin acts by activating this factor and preventing coagulation
c. Protein C and S- require vitamin K for -decarboxylation. Act
together to inactivate cofactors Va and VIIIa. Protein C is activated
by binding of thrombomodulin to thrombin. Protein S is a cofactor
for protein C. Prevents coagulation.
d. Plasmin- Inactive plasminogen gets incorporated in the developing
clot. Once it is activated it turns into plasmin which is proteolytic. It
degrades fibrin to fibrin degradation products. (Dissolutes the fibrin
clot)
v.

49

12. Identify the role of bleeding time, clotting time, prothrombin


time (INR), and activated patial thromboplastin time (APTT) in
the differentiation of bleeding disorders.
a. Hemostatic function tests
i.
Bleeding time- test for the time taken from the initial injury to
the formation of the primary hemostatic plug. Bleeding time is
an indicator of primary hemostasis. Prolonged bleeding time is
an indicator of low platelet count or vWF deficiency or platelet
receptor defects.
ii.
Clotting time- time taken for the formation of the stable
hemostatic plug. Prolonged clotting time indicates defects in
the
coagulation
pathway.
Specific
defects
of
the
instrinsic/extrinsic pathways are indicated by prothrombin time
and activated partial thromboplastin time (APTT/PTT)
b. Tests for Coagulation Disorders
i. Prothrombin Time (PT)- tests the extrinsic and the common
coagulation pathways. International normalized ratio (INR).
Measures defects in tissue factor, VII, V, X, prothrombin and
fibrinogen (extrinsic)
ii.
Partial thromboplastin time- (APTT)- Tests intrinsic and common
pathways. Measures defects in VIII, IX, XI, XII, V, X prothrombin
and fibrinogen.
13. Discuss the biochemical basis of the following genetic
disorders of hemostasis: hemophilia (A and B), Von Willebrand
Disease,
Bernard-Soulier
Syndrome,
Thrombasthenia
of
Glanzmann and Naegeli.
a. Hemophilia- inherited X-linked recessive disorder. They have a
coagulation factor deficiency. Easy bruising, massive hemorrhage
after trauma and surgical procedures. Spontaneous hemorrhages,
particularly in the joints- hemoarthrosis. Clotting time is increased
and APTT is increased (intrinsic pathway). Type A (VIII) and Type B
(IX).
b. Von Willebrand Disease- Most common inherited bleeding disorder.
Instability of factor VIII. Similar features to hemophilia A, with
increased mucosal bleeding, increased post operative bleeding.
Bleeding time is prolonged, APTT prolonged and vWF levels are low.
c. Bernard-Souller Syndrome- Qualitative defect of platelets. Normal
platelet count but increased bleeding time. GpIB defect.
d. Thrombasthenia of Glanzman and Naegeli- Qualitative defect of
platelets. Normal platelet count but increased bleeding time.
GpIIb/IIIa defect.
e. Thrombocytopenia- Low platelet count and increased bleeding time.

50

Lecture: Liver function tests


1. List the important functions of the liver
a. Excretion of bilirubin
b. Synthesis of plasma proteins
c. Detoxification of ammonia
2. Interpret the values of the following laboratory tests in the
diagnosis, follow up and prognosis of a patient with liver disease
(acute hepatitis, alcoholic liver disease, cholestatic disease)
a. Serum (total, conjugated and unconjugated) and urine
bilirubin
1. In Patients with hepatitis, serum total, conjugated and
unconjugated bilirubin is increased. Damaged liver has a lower
than normal capacity for uptake and conjugation of bilirubin.
Conjugated bilirubin may also be elevated due to inflammation
because the liver cell swells and blocks the ductules.
2. In Cholestatic disease serum conjugated bilirubin is elevated. In
complete obstruction conjugated bilirubin is not excreted in the
bile and it regurgitates into blood and appears in the urine.
Urobilnogen is also not formed resulting in clay colored feces
(stercobilinogen cannot be formed either). Serum bilirubin can
estimate the extent of the damage but it may be difficult to
distinguish hepatocellular disorder from a cholestatic disorder
on basis of bilirubin itself.
b. Serum enzymes (ALT, AST, GGT, 5NT, LDH) Allow you to
distinguish hepatocellular or cholestasis
1. Hepatocellular- ALT and AST are enzymes related to amino
acid metabolism in the liver. Levels are raised in viral hepatitis,
drug induced hepatitis, and long standing obstructive jaundice.
Raised ALT is more specific for liver cell damage compared to

51

raised AST. In acute hepatitis ALT >>> AST. In long standing


(chronic) alcohol cirrhosis AST >>> ALT.
2. Cholestasis- ALP and GGT. GGT synthesis is induced by alcohol
and this enzyme is a marker of alcohol consumption. ALP and
GGT are both elevated in biliary stasis (Cholestasis). If biliary
tree is dilated there is extrahepatic cholestasis, if it is not
dilated then intra-hepatic cholestasis is suspected. ALP increase
with no GGT increase indicates that it is a non-hepatic cause.
Elevation in GGT and not ALP indicates recent alcohol use.
c. Serum proteins (albumin and globulin)
1. All plasma proteins except the -globulins are synthesized by
the liver. Albumin helps in maintenance of colloidal osmotic
pressure. Chronic lever disease is characterized by low serum
albumin levels. Hypoalbuminemia leads to ascites and edema
in patients. Decrease of albumin indicates long standing liver
disease. Serum -globulins are increased in cirrhosis.
d. Prothrombin time
1. Liver synthesizes factors V, VII, IX, X, prothrombin and
fibrinogen. Normal prothrombin time is 12-15 seconds.
Increased time indicates deficiency of clotting factors
synthesized by the livers. May be due to hepatocellular disorder
due to decreased synthesis of clotting factors. May also be
elevated due to cholestasis. Cholestasis results in an impaired
absorption of vitamin K due to reduced entry of bile acids into
intestine. This impairs post translational -carboxylation
modification of vitamin K clotting factors II, VII, IX and X.
e. Special tests in evaluating liver function: Serum ammonia,
AFP, 1- antitrypsin, ceruloplasmin, serum iron, transferrin
and ferritin
1. Serum ammonia levels contribute to the development of
hepatic encephalopathy and altered consciousness in patients
with end stage liver disease. Impairment of urea formation by
the liver and [blood ammonia] rises.
2. Serum alpha fetoprotein (AFP) is used as a tumor marker in
patients with liver cancer.
3. Serum iron, transferrin and ferritin is used in patients suspected
to have hemochromatosis
4. Serum ceruloplasmin- patients suspected to have Wilsons
disease (copper)
5. Serum 1-antitrypsin in patients suspected to have hereditary
disease of the enzyme. Leads to emphysema.
___________Read the Cases at the end of the lecture for these last
three___________
3. Compare and contrast the changes in liver function tests in
hepatocellular diseases and disorders associated with cholestasis

52

4. Differentiate between acute and chronic liver disease based on


liver function tests
5. Explain the biochemical mechanisms for the following symptoms
and signs in patients with liver disease edema, icterus, ascites,
encephalopathy, bleeding tendency
Lecture: Alcohol and xenobiotic metabolism in liver
1. Explain the basic mechanisms of drug metabolism.
a. Drug metabolism in the liver can add or expose a functional group
that makes molecule more polar for eventual excretion in the
kidney.
2. Define phase I and phase II reactions.
a. Phase I involves cytochrome P450 (CYP450) and is in ER
membranes. Cytochrome P450 enzymes are involved with phase I
which leads to hydroxylations or other reactions. Cytochrome P450
enzymes use molecular oxygen and need NADPH and a heme
group. Cytochrome P450 reductase and cytochrome P450 work
together.
b. Phase II follows in many cases phase I but some drugs are also
changed into more water soluble forms and occurs in the cytosol.
Phase II uses mainly UDP-glucuronic acid, PAPS for sulfation,
glutathione and amino acids.
3. Describe the role of cytochrome P450 enzymes in drug
metabolism and specify CYP2E1 and CYP 3A4. CYP stands for
Cytochrome P450
a. Some drugs are inactivated by CYP450. Some prodrugs need
activation by CYP450 in order to be in their active form. Some drugs
may be converted to a toxic metabolite following the action of
CYP450.
b. CYP3A4 is one third of CYP450 in the liver and acts on more than
half of the therapeutic drugs.
c. CYP2E1 is specific for ethanol metabolism and is part of the MEOS
system in the liver.
4. Describe the mechanism of acetaminophen toxicity and the
biochemical basis for its management. (Page 42)
a. Acetominophen is designed to be mostly metabolized by the liver to
acetaminophen glucuronate and to acetaminophen sulfate in order
to be excreted by the kidney. If CYP2E1 is induced by chronic
alchohol abuse then this pathway can be increased and can lead to
a high amount of NAPQ I. Even then NAPQ I can be excreted by the
kidney after linkage to glutathione. At toxic high levels NAPQ I can
lead to cell death and sever liver damage especially when GSH is
not available in sufficient quantity for detoxification. Patients with
acetaminophen poisoning can be trated by administration of Nacetyl-cysteine (acetadote). This drug binds to NAPQ I and makes it

53

5.

6.

7.

8.

more water soluble and the cysteine part of the drug allows
synthesis of more GSH.
Describe the routes for ethanol metabolism. Discuss the role of
alcohol dehydrogenase, acetaldehyde dehydrogenases, and the
microsomal ethanol oxidizing system in the metabolism of
ethanol. (P 43)
a. Uptake of ethanol starts in the stomach. The liver is the major organ
to metabolize most of the blood ethanol to acetaldehyde and then
to acetate. Acetaldehyde is toxic and can be formed by alcohol
dehydrogenase and at very high ethanol concentrations by MEOS
and small amount by catalase.
b. At low alcohol levels in the liver Alcohol dehydrogenase converts
ethanol to acetaldehyde and creates an NADH. Alcohol
dehydrogenase have a low Km for ethanol.
c. At high ethanol levels the MEOS system which also generates
cytosolic acetaldehyde is also active. MEOS has a large KM and this
also creates radicals that cant be scavenged.
d. Acetylaldehyde that is formed in cytosol enters mitochondria.
Mitochondrial ALDH-2 has a smaller Km than cytosolic ALDH-1.
These reactions convert the toxic acetaldehyde to acetate and form
an NADH.
Outline the fate of acetate formed from ethanol
a. The acetate is released into the blood by the liver. The acetate is
used mainly in muscle and heart to form acetyl-CoA catalyzed by
acetyl-CoA synthetase. This is an irreversible reaction and the
formed acetyl-CoA enters the TCA cycle.
Discuss the physiological relevance of induction of CYP2E1 by
ethanol.
a. CYP2E1 has a lower affinity of ethanol than liver alcohol
dehydrogenase. This means you need a large amount of alcohol to
induce this enzyme. This enzyme generates more radicals than
normal in the cytosol. In addition the high acetaldehyde levels
caused by the large intake of alcohol leads to acetaldehyde binding
to glutathione and impairs scavenging of ROS. The increase in
radical formation by MEOS (CYP2E1) leads to damage of
mitochondrial DNA and proteins and general cell damage.
Describe the biochemical basis for the acute and chronic toxic
effects of ethanol abuse.
a. In chronic alcoholic individual, the high NADH levels lead to
increased synthesis of TAGs and VLDL formation. This happens after
a meal and also during fasting. Lactate cannot be used to form
pyruvate and the lactate stays in the blood. Pyruvate is lost as
lactate. Glucogenic amino acids cannot be used as OAA cannot be
formed from malate in cytosol. Glycerol-3-P cannot be used to form
DHAP in cytosol.

54

9. Discuss the treatment of poisoning by methanol or ethylene


glycol using ethanol.
a. Both poisons are substrates in cytosol for alcohol dehydrogenase.
Treatment can include in both cases inhibition of alcohol
dehydrogenase by ethanol or a competitive drug (fomepizole).
b. Methanol poisoning can be treated using ethanol or medical drugs
to compete with methanol as substrate for alcohol dehydrogenase.
Alcohol dehydrogenase has a higher affinity for ethanol than for
methanol. Methanol can lead to formaldehyde which can result in
blindness in severe cases.
c. Ethylene glycol is an alcohol found in antifreeze and when ingested
alcohol dehydrogenase forms glycoaldehyde. Can lead to kidney
failure and death due to formation of calcium oxalate. Treatment
includes emptying of stomach and patients need to be on ICU>
10. Describe the action of drugs that inhibit aldehyde
dehydrogenase (Disulfiram) (Page 47)
a. Acetaldehyde dehydrogenase, especially the mitochondrial form, is
inhibited by disulfiram (antabuse). The levels of acetaldehyde
increase as cytosolic acetaldehyde dehydrogenase has a larger Km.
Acetaldehyde can accumulate to levels that lead to flushing and
vomiting. Disulfiram treatment should prevent intake of even low
amounts of alcohol in these patients, but it can be also dangerous
when the patient drinks large amounts of alcohol (in spite of
disulfiram) and the acetaldehyde levels can increase dramatically to
toxic levels.
Lecture: Intertissue relationships: Adipose Tissue, Muscle and Brain
Metabolism (2 lectures)
1. Develop the concept that the metabolism of the body is
influenced by the liver, adipose, muscle and brain, as well as
other tissues (This is dumb Im not answering it)
2. Recognize the 5 major functions of adipose tissue and
differentiate between the two general types of adipose tissue
(white and brown)
a. Energy storage (Discuss TAG as a concentrated form of
metabolic energy)
b. Protection of internal organs
c. Insulation of body heat
d. Adipose as an endocrine gland
i. Discuss leptin as an adipokine- Leptin receptors are in the
hypothalamus. Product of the Ob gene (obesity) and tells brain
youve eaten enough. Leptin induces expression of UCP-2.
Uncoupling proteins are found in the inner mitochondrial
membrane. UCP form channels which allow dissipation of the

55

3.

4.

5.

6.

proton gradient. Contributes to heat production. Leptin also


activates AMPK (AMP-dependent kinase).
ii. Discuss leptins role in appetite regulation- Mediates
appetite suppression.
iii. Link leptin resistance to insulin resistance- In obese
patients leptin levels are high and there is no signal pathway to
stop the person from eating. The signaling pathway is messed
up. Likewise in insulin resistance there is an absence of signal
response so it results in high insulin after a meal. The high
insulin is constant because the person keeps eating and the
leptin is being produced to tell the person to stop, however the
receptors are defective so no signal is being sent.
e. Thermogeneration (Brown Adipose Tissue, BAT)
i. Discuss the cellular characteristics of BAT- more
mitochondria and many dispersed fat droplets.
ii. Function of the H+ uncoupler thermogenin (UCP1)- Allows
thermogenesis and works by dissipating the proton gradient in
the mitochondria. Forms pores in the membranes.
Review TAG hydrolysis and the synthesis of TAG in adipose tissue
a. Relate to the fed/fast state- Fasting state hormone sensitive
lipase will be activated by epinephrine and glucagon. There is a
decrease in insulin. In the fed state HSL will be inhibited by insulin.
b. Review sources of FA for the adipose cell: Chylomicrons and
VLDL- Occurs in a well fed state. High insulin/glucagon ration leads
to an increase in GLUT-4 transporters. Glucose uptake into the liver.
VLDL is synthesized in the liver and released into the blood.
Lipoprotein lipase of adipocyte is activated by insulin. Cleaves TAGs
in VLDL and CM (Apo-C-II).
Review the possible sources of glycerol-3-phosphate in the
adipose and the liver and discuss the fate of the glycerol that is
released from adipose tissue TAG lipolysis
a. Glycerol that is released from the adipose tissue by TAG lipolysis
goes to the liver where is converted to glycerol 3-P by glycerol
kinase which can be converted to DHAP Gluconeogenesis. In the
adipose tissue Glucose is taken in by the cell and converted to
DHAP which is converted to Glycerol 3-P and is used in TAG
synthesis.
Associate abdominal fat with increased risk of coronary heart
disease (Lippincotts 5th Edtion p350)
a. Visceral fat is located inside the abdominal cavity packed in
between the internal organs. This fat can compress the thin arteries
that travel to the heart causing CHD.
Differentiate between Type I (red), Type IIa, and Type IIb (white)
muscle fibers

56

a. Present the general structure of a muscle fiber including a


description of the dystrophin protein- Dystrophin protein
(Couldnt find it in lecture)
Type I slow
Type IIA Fast
Type IIB Fast
Property
twitch
oxidative
Glycolytic
ATP Hydrolysis
Slow
Fast
Fast
Contraction
Slow
Fast
Fast
Speed
Glycolytic
Low
Moderate
High
Capacity
Oxidative
High
Moderate
Low
Capacity
Glycogen
+
++
+++
Storage
Appearance
Red
Red
White
Capillary
Good
Moderate
Poor
Supply
7. Compare energy metabolism in resting muscle and exercising
muscle tissues
a. Discuss fuel sources and relate to the fed/fast state- At rest
muscle uses ~30% of oxygen. During vigorous exercise it uses
~90% of oxygen. In fed state in creased glucose transport via
GLUT 4. During starving state you have an increase in FA uptake. In
resting muscle high ATP levels drive the synthesis of creatine
phosphate. In exercising muscle low ATP levels allow creatine
phosphate to drive the synthesis of ATP from ADP.
b. Discuss the changes that occur during long duration
exercise- Prolonged exercise of starvation the Blood glucose goes
down, so that is a decrease in insulin and an increase in glucagon.
This leads to an increase in FA usage. HSL is activated in adipose.
Insulin dependence on glucose uptake may be overcome by
elevated AMP levels. Signals through the AMP-protein kinase (AMPK)
mobilizes GLUT 4 transporters to sarcolemma.
8. Differentiate aerobic and anaerobic skeletal muscle metabolism
a. Discuss how creatine-phosphate functions as an ATP bufferIn resting muscle it stores ATP because ATP is high energy and is
used quickly, dont want to waste it in a resting muscle. When
muscle needs ATP in converts ADP ATP.
b. Discuss how fuel sources change in anaerobic muscle
contraction- Lactate produced is used via the Cori cycle, muscle or
heart. This happens when ATP requirements exceed aerobic
capacity.
c. Discuss the end electron acceptor during energy generation

57

i. Oxygen in aerobic exercise and lactic acid formation


from pyruvate
ii. The central importance of oxidizing NADH to NAD +Generates NAD+ for Ketone body synthesis. The Krebs cycle
also needs NAD+
d. Review the regulation of glycogen degradation
e. List the biological fuels that cardiac muscle may use under
normal conditions- Cardiac muscle may use any fuel. Always
dependent on vascular supply.
9. Review the concept of the blood brain barrier (BBB) in relation
to brain metabolism
a. Fatty acids from TAG mobilization may not cross the BBB
(dietary essential fatty acids may enter the brain)- Small
uncharged molecules and non-polar substances freely pass.
b. Dietary essential amino acids may cross the BBB
c. Explain the BBB metabolic block to L-DOPA Amino acids that
are NT are blocked from entry to the brain to separate the somatic
pools from the neural pools. This way if the brain needs a
neurotransmitter itll just make it. Treatment of parkinsons is using
L-DOPA. L-DOPA can cross the blood brain barrier but it will be
quickly degraded by neuronal endothelial cells. DOPA decarboxylase
inhibitor must be given in combination with L-DOPA for efective
therapy.
10. Discuss the nutrient molecules that are used for energy
production in the CNS
a. The brain uses Glucose and this is not insulin responsive. Ketone
bodies are used in the brain and during starvation KB transporter in
the brain is upregulated.
11. Compare the fed state to the changes that occur after
prolonged fast
a. In the well fed state the diet supplies the energy requirements for
the brain via glucose. In the fasting state, glycogenolysis and
gluconeogenesis maintain glucose levels to supply brain with fuel.
Under normal conditions there is abundant glucose in circulation.
During a prolonged fast there is significant rise in serum ketone
bodies. Ketone bodies become the major brain fuel
12. Identify the central role of glutamate in neurotransmission:
a. Glutamate as a major excitatory neurotransmitter- Receptors
found in post-synaptic cell. Many subtypes named for drugs that
bind the receptor. Glutamate signal termination occurs by reuptake
in presynaptic terminals or by glial transporters.
b. GABA as a major inhibitory neurotransmitter- Epilepsy is
associated with low GABA. Epilepsy may be treated with GABA
analogs or drugs that block GABA reuptake from synapse.
c. Describe GABA metabolism in the brain, including the GABA
shunt- GABA is recycled in the CNS GABA shunt. Conservation of
58

glutamate and GABA. Glial cells contribute the major uptake of


GABA. Glial cells lack the glutamate decarboxylase. GABA shunt
converts GABA into TCA cycle intermediate so it can go back to ketoglutarate so it is essentially being recycled.
13. Describe the metabolism of acetylcholine in the brain
a. Dependence on SAM and vitamin B 12- Required to regenerate
SAM to form choline. Needed for neurotransmitters (ACH).
b. Action of acetylcholine esterase- Cleaves acetylcholine
14. Describe the structure and function of myelin and link it to the
destruction of myelin that is observed in a patient with Multiple
Sclerosis
a. Myelin is a multi layered membrane sheath made up of lipids and
proteins with high content of sphingomyelin and cerebrosides
(Speeds action potentials). Multiple sclerosis is characterized by a
progressive destruction of CNS myelin. Formation of sclerotic
plaques which slows neurotransmission (leads to eventual loss). An
autoimmune reaction: possible triggered by a viral infection.
Episodal periods of destruction and symptoms are weakness, lack of
coordination, loss of vision.
15. Compare and contrast the biochemical mechanisms underlying
the neurodegenerative disorders of Alzheimers and Prion
diseases
a. Alzheimers is progressive loss of memory and cognition.
Mitochondrial dysfunction in neural cells. Results from the
inappropriate accumulation of proteolytic fragments from the amyloid precursor protein. The neurotoxic peptide has a high sheet content forms aggregates and neurofibrillary tangles.
b. Prion Disease- Changes in tertiary structure of the PrP c protein.
Same thing as Transmissable Spongiform encephalopathy disease.
Prions are proteins that may act as infectious agents when
incorrectly folded. Normal prion protein (PRP) is soluble and is found
on the extracellular side of neuronal membranes. The defective one
has -sheet and is resistant to protease digestion.
16. Revise the importance of vitamin B1 on brain metabolism
(Reading assignment)
a. Wernicke Korsakoff syndrome and Beri-beri- Moderatively
severe thiamine deficiency (B1), mental confusion, ataxia,
opthalmoplegia. Most common in chronic alcoholics and responds to
thiamine supplement.
17. Pathways of the biosynthesis of Catecholamines: dopamine,
norepinephrine, epinephrine derivatives of tryptophan: Serotonin
and melatonin (This is on other lectures)
Lectures: Vitamins and Minerals I and II (2 lectures)
1. List and group the major vitamins of the human diet (water
soluble and lipid soluble)
59

a.

Water Soluble- Vitamin C (Ascorbic Acid), Thiamine (B1),


Riboflavin (B2), Niacin (B3), Biotin, Pantothenic Acid, Folic Acid,
Vitamin B12, Pyridoxine (B6), Pyridoxal, and Pyridoxamine
b.
Fat-Soluble (Require Bile for Absorption)- Vitamin A, D, E, K (we
do not store large amounts of vitamin K.
2. Discuss the significances of vitamin B and vitamin C deficiencies
a.
Vitamin C
i.
Scurvy- fragile blood vessels, sore and spongy gums,
bleeding into joints, impaired wound healing. Defects in
connective tissue because of decreased hydroxylation of
collagen. Less stable collagen. Can be caused by a lack of fruits
and veggies in the diet.
3. Indicate the coenzyme forms, the reaction requiring and the
biochemical basis of clinical manifestations of deficiency of
thiamine (B1), riboflavin (B2), niacin (B3), and pyridoxine (B6)
a.
Vitamin B1 (Thiamine)- Oxidative decarboxylation of alpha keto
acids. Needed for pyruvate dehydrogenase, -ketoglutarate
dehydrogenase, and branched chain -ketoacid dehydrogenase.
Acts as a coenzyme for transketolase in the HMP pathway. Polished
rice, white flour and white sugar are deficient in TPP.
i.
Beri Beri- When polished rice is major diet component.
Disruption of motor, sensory and reflex arcs. Neurological
manifestations is dry beri beri and cardiovascular symptoms is
wet beri beri.
ii.
Wernicke-Korsakof Syndrome- Associated with chronic
alcoholism. Ophthalmoplegia and nystagamus (to and fro
movement of eyeballs).
b.
Vitamin B2 (Riboflavin)- Forms FMN and FAD and participates in
oxidation reduction reactions of TCA cycle, Beta oxidation. Succinate
DH, Pyruvate DH ,and Acyl CoA DH (MCAD)
i.
Cheilosis- areas of pallor cracks and fissures at the angles
of the mouth.
ii.
Glossitis- inflammation of the tongue
iii.
Facial Dermatitis
c.
Vitamin B3 (Niacin)- Coenzyme forms NAD+ and NADP+. Niacin
inhibits lipolysis in adipose tissue and greatly reduces production of
free fatty acids.
i.
Pellagra- rough skin characterized by the 3 Ds (dermatitis,
diarrhea and dementia). Dermatitis is exposed areas of body,
redness, thickening and roughening of skin. You will see people
with necklace like skin damage and eventually leads to death.
Tryptophan can be used to synthesize NAD+ and NADP+. Corn
based diets can cause pellagra because its deficient in Niacin
and Tryptophan.
d.
Vitamin B6 (Pyridoxine)- Serve as precursors for PLP which acts
as coenzyme for transamination, amino acid decarboxylation (dopa60

decarboxylase, serotonin, and glutamine decarboxylase), ALA


synthase
and
conversion
of
homocysteine
to
cysteine
(Cystathionine -Synthase).
i.
Deficiency leads to mycrocytic anemia (ALA synthase) and
increased risk of cardiovascular disease because of increased
levels of homocysteine.
ii.
Isoniazid is an antituberculosis drug that inactivated
pyridoxine.
e.
Vitamin B12 (Cobalamin)- Essential for synthesis of methionine
and isomerization of methylmalonyl CoA in odd number fatty acid
degradation.
In
B12
deficiency
folate
is
trapped
as
methytetrahydrofolate and results in macrocytic anemia. Lack of IF
results in a poor absorption of B12 and pernicious anemia.
i.
Significant amounts stored in the body and symptoms
include macrocytic anemia. Folate is trapped and not available
for purine/pyramidine synthesis. You get neuropsychiatric
symptoms. Myeline degradation in both motor and sensory
pathways due to methylmalonyl CoA accumulation. (If folate is
deficient you will not get neurological symptoms).
1. Folate is acquired from the diet through green leafy
vegetables. Can also be due to cobalamin deficiency
(folate trap). Megaloblastic anemia has to be treated
with folic acid and B12.
4. Describe the absorption of fat-soluble vitamins. Predict the
reason for occurrence of fat soluble vitamin deficiency in patients
with fat malabsorption
a.
The absorption of fat soluble vitamins is dependent on normal fat
digestion and absorption. Maldigestion and malabsorption of dietary
fats results in secondary deficiency of the fat soluble vitamin (bile
duct obstruction, cystic fibrosis). The fats arent being incorporated
into chylomicrons so neither are the vitamins.
5. Describe the use of vitamin K as coenzyme for -carboxylation of
inactive blood clotting factors in the liver.
a.
Required post translational modification of various clotting
factors (II, VII, IX and X). In humans it is also synthesized by
bacterial flora. Vitamin K serves as a cofactor for liver microsomal carboxylase. Vitamin K dependent -carboxylation of glutamic acid
residues. The addition of another carboxyl group on respective
glutamate residues leads to a renaming from Glu residues to Gla
residues. -carboxylation allows formation of a mature clotting
factor that is capable of subsequent activation. -carboxylation
allows Calcium binding because of two adjacent negatively charged
carboxylate groups. The clotting factor calcium complex can then
bind to phospholipids on the platelet membrane.
6. Discuss hemorrhagic disease of the newborn. Predict the causes
and effects of vitamin K deficiency.
61

a.

The newborn has a sterile intestine and vitamin K 2 is not formed


by bacteria. Breast milk as natural nutrition for the newborn is low
in vitamin K1. The enzymes for the synthesis of inactive blood
clotting factors and for -carboxylation in the liver have still low
activities in the newborn which will increase in the following days
and weeks. Routine intramuscular injection of vitamin K is
recommended for all newborns.
7. Discuss inhibition of -carboxylation by warfarin and compare and
contrast the anticoagulant action of heparin and warfarin.
a.
Warfarin interferes with -carboxylation of inactive blood clotting
factors in the liver. It blocks the activity of liver epoxide reductase
and prevents regeneration of reduced Vitamin K. This leads to a
delay in clotting. These drugs do not have a short-term efect on
blood clotting and are not used for prevention of blood clotting
during surgery.
b.
Heparin is the anticoagulant of choice to reduce blood clotting
during surgery. Heparin facilitates the binding of anti-thrombin III to
thrombin, and with that it limits the action of thrombin.
8. Describe the deficiencies of dietary vitamin A and retinoids and
discuss use of retinoids as drugs in medicine
a.
Collectively called retinoids. Dietary deficiency is the commonest
cause fat free diets and malabsorption of fats can also lead to
deficiency. Signs and symptoms may include night blindness
(Earliest), xerophthalmia (dryness of cornea), Bitots spots (white
spots in cornea), Keratomalacia (softening/erosion of cornea),
increased risk of pulmonary infections and weakened immune
system.
b.
Retinoic acid is used to treat severe acne and psoriasis. Too much
can be toxic leading to Hypervitaminosis A (Headaches), dry and
pruritic skin, enlarged liver, and spontaneous abortions in
pregnancy. Vitamin A should be avoided in pregnancy.
9. Discuss the formation and action of retinoic acid
a.
Retinol enters the target cell and is oxidized to retinoic acid in
the cytosol. From the cytosol, the retinoic acid moves into the
nucleus with the help of cellular retinoid binding proteins. Retinoic
acid binds to nuclear receptors forming an activated receptor
complex. Retinoic acid-receptor complex binds to chromatin
activating the transcription of specific genes (keratin).
10. Describe the function of cis-retinal in vision. Indicate why
retinoic acid cannot be used to treat night blindness.
a.
Retinol is transported to the retina and enters the retinal pigment
cells. 11-cis retinol is oxidized to 11-cis retinal. 11-cis retinal enters
the rod cell where it combines with opsin to form rhodopsin (visual
pigment). Absorption of a photon of light catalyzes the
isomerization of 11-cis-retinal to all-trans-retinal triggering a

62

cascade of events, leading to the generation of an electrical signal


to the optic nerve which is interpreted as vision.
b.
Retinoic acid cannot be used to treat night blindness because it
cannot be converted into retinol or retinal. The pigment cells need
retinol/rods need retinal.
11. Describe the formation of vitamin D in the skin and the
formation of calcitriol (role of liver and kidney) (Draw page 24
slide 19)
a.
7-dehydrocholesterol is converted to cholecalciferol by sunlight.
This cholecalciferol goes to the liver where it is hydroxylated by 25hydroxylase to 25-hydroxycholecalciferol. This molecule then goes
to the kidney where it is hydroxylated again by 1-hydroxylase to
Calcitriol. (1-hydroxylase is the RLS stimulated by parathormone
and a decrease in calcium) 1,25 dihydroxycholecalciferol is the
same thing has calcitriol.
12. Describe the functions of 1,25 dihydroxy-D related to calcium
metabolism
a.
1,25 Dihydroxycholecalciferol binds to intracellular receptor
proteins. 1,25-dihydroxy-D3 receptor complex interacts with DNA in
the nucleus of target cells (intestine). They can then selectively
stimulate gene expression or repress gene expression. It can
stimulate the intestinal absorption of calcium and phosphate by
increased synthesis of a specific calcium binding protein. On the
bone it can stimulate the mobilization of calcium and phosphate
form the bone in the presence of parathormone. On the kidneys can
inhibit calcium excretion by stimulation parathyroid dependent
calcium reabsorption. INCREASES PLASMA CALCIUM
13. Describe the biochemical basis of clinical manifestation in
rickets and osteomalacia.
a.
Vitamin D deficiency in children and it is due to vitamin D
deficiency in children. You have a decreased calcium absorption in
the diet increase in parathyroid hormone release increase in
demineralization of the bone. Bones become soft and pliable and
you get the characteristic bow leg deformity. You also get a pigeon
chest deformity. Frontal bossing.
b.
Osteomalacia is Vitamin D deficiency in adults. Bones are
demineralized and are susceptible to fracture.
14. Discuss the role of copper as cofactor for enzymes
a.
Important cofactor in redox reactions. Cytochrome C is part of
complex IV in the ETC. Superoxide dismutase, lysyl oxidase
(synthesis of collagen), and tyrosinase (neurotransmitter synthesis).
Copper forms ceruloplasmin in the liver, which apart being a copper
transport protein, helps in iron metabolism.
b.
Ingested copper is absorbed in the stomach and intestine and
transported to the liver via albumin. In the liver it is used to form
ceruloplasmin which is secreted into the plasma. Aged
63

ceruloplasmin is taken up by the liver from the plasma, endocytosed


and degraded and copper is secreted into bile.
c.
Signs and symptoms- microcytic anemia (smaller RBC) because
iron metabolism needs copper. Degradation of vascular tissue
because lysyl oxidase needs copper. Defects in hair.
15. Discuss clinical manifestations and biochemical defects of
defective copper metabolism in Wilsons disease and Menkes
syndrome.
a.
Menkes Syndrome (Kinky Hair syndrome)- Inherited defect in
absorption of copper from GI tract. Low levels of copper in plasma
and most tissues. Hair is twisty, grayish and kinky. Copper
deficiency can lead to aneurysms and cerebral dysfunction.
b.
Wilsons Disease- Accumulation of toxic levels of copper in vital
organs including liver, brain and eye. Defect is found in the copper
transporting ATPase in the liver. This protein is needed to attach
copper to ceruloplasmin and also to excrete copper into the bile. You
see characteristic Kayser Fleischer Rings in the cornea. Decrease
serum ceruloplasmin, increased urinary excretion of copper and
increased hepatic copper content. (P 36 of objectives to look at
ceruloplasmin)
16. Discuss the laboratory findings and biochemical basis for
clinical manifestations in hereditary Hemochromatosis.
a.
Required for heme synthesis and is important for redox reactions.
The dietary iron absorption is tightly regulated by body iron stores.
Higher the body iron stores less iron is absorbed by the intestine.
Iron needs to be in ferrous state to be absorbed and the stomach
converts ferric iron to ferrous iron. Ceruloplasmin (ferroxidase)
participates in the release of ferrous iron from intestinal cells and
forms ferric iron which is needed for transport in the blood.
Transferrin is transport protein for ferric ion in blood plasma.
b.
Most common nutritional deficiency. Hemochromatosis is
excessive absorption of iron. HFE gene in intestine regulates iron
uptake and it is defective in this disorder. Unregulated uptake of
iron. More common in males and iron damages tissues by lipid
peroxidation through free radicals and DNA damage. Also you see
liver damage, diabetes (destruction of pancreas), cardiac
dysfunction and brownish pigmentation of the skin.
17. Indicate the nutritional causes of microcytic anemia.
a.
Microcytic anemia is the result of reduced heme synthesis in
erythroid cells. This can be due to deficiency of pyridoxine which
leads to deficiency of PLP needed as coenzyme for ALA synthase.
[Glycine is decarboxylated in this step]. It can also result from
deficiency of iron leading to less heme synthesis in erythroid cells.
[Copper deficiency leads to iron deficiency]. Lead toxicity leads to
microcytic anemia, as lead inhibits ALA dehydratase and

64

ferrochelatase and less hemoglobin than before is formed in


erythroid cells.
Lecture: Introduction to nutrition
1. Analyze the components of daily energy expenditure
a. Resting energy requirement (REE/ RMR/ BMR) and
enumerate factors affecting it
i. REE- resting energy expenditure
ii. RMR- Resting Metabolic Rate- can be efected by gender, body
temperature, environmental temperature, thyroid function,
pregnancy and lactation and age.
iii. BMR- Basal metabolic rate- energy to support our metabolism
b. Physical activity- Sedentary you have 30% expenditure of RMR,
moderate activity you have 60-70% expenditure of RMR, and
several hours of heavy exercise you have 100% expenditure of RMR.
c. Diet induced thermogenesis (SDA)- Represents the energy
required to digest, absorb, distribute and store nutrients.
2. Differentiate the macronutrient ratios of a balanced diet, energy
content of macronutrients and list important sources of carbohydrates, proteins, fats, ethanol and dietary fiber
a. Carbohydrates (45-65%)- Provide 4 kcal/g. Abundant in fruits, sweet
corn, corn syrup and honey. Polysaccharides found in wheat, grains,
potatoes, dried peas, beans, vegetables, starch and fiber (no energy
but adds bulk to diet).
b. Fat (20-35%)- Provide 9 kcal/g. Corn oil, soybean oil, olive and
canola oil, margarine. You want more saturated fats and increase
unsaturated (ex- TAGs from plants).
c. Protein (10-35%)- Provide 4 kcal/g. Meat, poultry, milk, and fish.
d. Ethanol- Provides 7 kcal/g.
3. Indicate the significance of dietary carbohydrate and predict the
significance of glycemic index of carbohydrates
a. Some carbs (high glycemic index) will raise serum glucose more
rapidly than others and the decline in serum glucose will also be
steeper. Needs carbs for glucose.
4. Determine the significance of dietary fats in relation to coronary
heart disease, and essential fatty acids
a. Provide EFA linoleic and linolenic acids. EFAs required for membrane
fluidity and synthesis of Eicosanoids. Deficiency of EFA
characterized by scaly dermatitis, hair loss, and poor wound
healing. CHD = blockage of heart.
5. Explore the significance of dietary fiber and correlate it to clinical
applications of dietary fiber- Consists of non-digestible carbs. Included
cellulose, lignin, and pectin. Adds bulk to diet and increases bowel
motility. Decreases risk for constipation, hemorrhoids, diverticulosis and
colon cancer.
65

6. Analyze the significance of dietary protein and differentiate


between the sources of high and low quality protein. Indicate
states of positive and negative nitrogen balance
a. Protein is needed to replenish body proteins. 20 are essential and
the high value sources contain high amounts of EAA. Gelatin, wheat,
corn, rice and beans are low biological value sources of protein.
b. Positive nitrogen balance is when N intake > N excretion (tissue
growth)
c. Negative nitrogen balance is when N excretion > N intake
(inadequate dietary protein, stress)
7. Explore the roles of various hormones involved in appetite
regulation (leptin, insulin, ghrelin)
a. Leptin- a peptide produced by adipose tissue. Leptin is produced
proportionally to fat cell density and plays a key role in weight gain,
appetite suppression and energy expenditure through its action on
the hypothalamus. It also regulates inflammatory responses, blood
pressure and bone density
b. Insulin- similar to leptin, dampens appetite by exerting its efects on
the hypothalamus.
c. Ghrelin- a peptide hormone secreted by the stomach that increases
the release of neuropeptide Y. This enhances appetite and food
intake. Ghrelin release is activated by fasting and low glucose and
inhibited by high glucose. CCK and PYY cause satiety during a meal
and transmit those signals to the hypothalamus.
8. Solve problems involving calculation of energy expenditure in an
individual (based on activity)
9. Solve problems involving dietary macronutrient ratios for a
balanced diet
10. Distinguish the methods to assess nutritional status
(anthropometry and biochemical)
a. Anthropometry- BMI provides a measure of relative weight adjusted
to height. Healthy range 18.5-24.9. Overweight 25-29.9. Obese is
greater than 30. Mid-arm skin-fold thickness is used to assess
subcutaneous fat by comparing it to population standards.
Lecture: Obesity
1. Definition of obesity in terms of BMI
a. Obesity is when BMI = Weight (kg)/Height (m2) is greater than
30kg/m2
2. Analyze methods for diagnosis of obesity
a. Compare the various methods available for diagnosis of
obesity: BMI 30, waist circumference ( 40 for men, 35
female), Waist to hip ratio ( 0.9 for men, 0.8 for women),
bioelectric impedance (measures conduction speed of a small
electric current in the body)
66

3.

4.

5.

6.

b. Discuss the significance of location of body fat (apple vs


pear) in obesity- apple is in the abdomen (visceral) and pear is
in the thighs (subcutaneous).
Summarize the factors contributing (multifactorial) to obesity
a. Genetic- Mutations in the leptin gene or its receptor can produce
hyperphagia and massive obesity
b. Environmental and behavioral factors- Lifestyle and behavior
c. Explore the role of leptin in obesity- Previous lectures. Leptin is
supposed to suppress appetite. If you have a defect what happens?
Analyze the metabolic changes in obesity
a. Dyslipidemia Correlate biochemical changes in lipid profile
to metabolic changes in obesity- Too much lipids in the blood so
that leads to an increase in serum TAGs and VLDL and fat
deposition.
b. Syndrome X (Metabolic syndrome) Correlate clinical and
biochemical features and explain the significance of insulin
resistance- Extra weight around your waist leads to increased
visceral fat and higher chance of CHD. You also have low levels of
HDLs and insulin resistance. Results in increased TAGs in circulation
due to increase synthesis of VLDL by the liver and results in
increase risk of atherosclerosis. Insulin resistance is correlated with
weight gain.
Explain the biochemical basis for management of obesity
a. Decrease caloric intake and increase physical activity. Improve
quality of food consumed and increase intake of fruit and vegetable
(increase fiber).
Determine the role of drugs (sibutramine and orlistat) and
surgery in obesity management
a. Sibutramine- an appetite suppressant (increases fullness- a feeling
of satiety)
b. Orlistat- gastric and pancreatic lipase inhibitor, inhibits the digestion
of dietary fats (TAGs) and therefore reduces dietary TAG absorption.
c. Surgery- Recommended for morbidly obese BMI > 35. Decrease size
of stomach, ileo gastric bypass. They are used in conjunction with
behavioral modifications to achieve long-term weight loss
objectives.

Lecture: Starvation and Undernutrition


1. Summarize hormonal, metabolic changes and changes in the
various organs (liver, muscle, brain, adipose tissue) in the various
phases listed below
a. Postprandial phase (2-3 hours after a meal) Glucose and
insulin levels increase in blood after a meal
b. Post absorptive phase (5-7 hours after a meal) - Usage of
glucose leads to lower blood glucose levels. Insulin/glucagon ratio
67

2.
3.

4.

5.

changes in favor of glucagon. Liver metabolism is characterized by


release of glucose into the blood using glycogen degradation and
gluconeogenesis. Release of FFA into the blood.
c. Early phase of starvation- Plasma levels of Fatty acids and
ketone bodies increase while glucose decreases.
Prolonged starvation adaptations to increase survival, role of
kidney- Kidney performs gluconeogenesis.
Predict the deleterious effects of prolonged starvation- Substantial
loss of body fat and muscle mass, vitamin and mineral deficiencies reduce
enzyme activities, atrophy of intestine, diarrhea, skin rashes, edema,
hypothermia and heart failure.
Correlate important clinical features with biochemical basis for
anorexia nervosa
a. Patients refuse to maintain normal body weight as they feel fat even
when undernourished. Related to low self esteem. They have low
blood pressure, bad memory, fatigue, anemia, weak muscles,
kidney failure, constipation and loss of menstrual cycle. Associated
with Marasmus
Regarding protein energy malnutrition
a. Distinguish and define types of PEM- Protein energy
Malnutrition leads to variable clinical conditions with extreme forms,
including patients with major trauma, and depressed immune
system.
b. Compare and contrast features of marasmus and
kwashiorkor
i. Marasmus- Chronic dietary restriction of carbs, lipids, proteins
and other nutrients. Extensive tissue and muscle wasting. Dry
skin, loose skin folds hanging over buttocks. Drastic loss of
body fat on buttocks and thighs. Associated with anorexia
nervosa.
ii. Kwashiorkor- Deficiency of dietary protein and leads to muscle
wasting due to lack of EAA. Decreased serum albumin so you
have edema in abdomen and legs.
c. Correlate biochemical basis for the clinical features of both
(above)
d. Identify associated vitamin and mineral deficiencies in PEMVitamin deficiencies can lead to beri-beri (B1), cheilosis (B2),
pellagra (B3), scurvy (Vitamin C), anemia (B12), reduced blood
clotting (Vitamin K), reduced response to infections and poor wound
healing.

68

Lecture: Metabolic response to trauma


1. Generalize the features of the hypermetabolic response and
enumerate its causes
a. Increase in glucagon, cortisol, ADH, Glucocorticoids, Insulin
(However you have insulin resistance). Increased catabolism of
peripheral protein in skeletal muscle and lipolysis because of
phagocytic release of mediators. Because of loss of fluid and
electrolytes you have the release of catecholamines, aldosterone,
ADH and glucocorticoids. Through Alanine-glucose cycle you use
amino acids for gluconeogenesis and synthesis of acutely needed
proteins like C-reactive protein.
2. Correlate hormonal and metabolic changes (carbohydrate,
protein and lipid) following trauma in the three phases.
a. Ebb phase (Unresuscitated phase)
i. Ebb phase- decreased cardiac output, O2 consumption, and
body temperature. This is the immediate response following
an injury. You can have lactic acidosis due to decreased tissue
oxygenation (decreased cardiac output). Insulin levels drop
this is diferent from the flow phase. Carbs- you have low
insulin levels with slight increase in glucose production.
b. Flow phase (Adrenergic corticoid phase)
i. Increased cardiac output, O2 consumption, body temperature,
energy expenditure and protein catabolism. Marked increase
in glucose production, increase in free fatty acids because HS
lipase is active because the body is not responding to insulin
and catecholamines are activating it (No ketogenesis). You
have an increase of catecholamines, insulin, glucagon, and
cortisol. Amino acids are used for the synthesis of acute
phase proteins. Carbs- you have high blood glucose, with
high insulin (resistance), decrease uptake of glucose,
increase in gluconeogenesis. Glucose is used by the injured
tissue and converted to lactate leading to lactic acidosis.
Protein- Negative nitrogen balance and skeletal muscle is the
major source of the N2 loss following injury. Glutamine and
Alanine are majority of the released Amino acids. They are
used for gluconeogenesis or acute phase protein synthesis.
c. Recovery phase (convalescent/ anabolic phase)- Not in lecture
69

3. Indicate the significance of acute phase proteins in response to


trauma
a. Measurement of acute-phase proteins, especially C-reactive protein,
is a useful marker of inflammation
4. Explore the significance of nutritional support and role of
glutamine supplementation
a. Glutamine is given for enhancement of immune function.
Supplementation is reported to improve immune functions (this is
everything thats on the slide about this)
5. Distinguish the pros and cons of enteral vs parenteral nutrition
support
a. Enteral Nutrition- via the gut and preserves intestinal mucosal
integrity better than parenteral nutrition with bowel rest. You have
decrease in mortality, bacterial translocation to mesenteric lymph
nodes, decrease in sepsis, and it preserves GI flora better. A
prerequisite is that you have a functional GI tract.
b. Parenteral nutrition is not as good and it is through IV and it
bypasses the gut. This is not good because it doesnt preserve the
intestinal mucosal integrity.
6. Distinguish the metabolic response to stress and starvation
Simple Starvation
Severe Injury
BMR

Inflammatory
+++
Mediators
Major Fuel
Fat
Protein + Fat
Ketone Bodies
+++
Negative N2 balance
+
+++
Blood Glucose Level
Lower

Gluconeogenesis and
+
+++
Proteolysis
Hepatic Protein
+++ (C-Reactive
+
Synthesis
Protein)
Lecture: Diabetes mellitus
1. Define diabetes mellitus
a. Group of disorders characterized by the presence of hyperglycemia
that result from defects in secretion of insulin OR action of insulin
OR both
2. Distinguish between the types of diabetes: type I and type II
(differences between the two types in terms of insulin)
a. Type I diabetes is insulin dependent diabetes and it is autoimmune
destruction of the -cells of the pancreas and marked decrease in
insulin secretion. Patients have to be on life long insulin
supplementation to prevent complications of diabetes.

70

b. Type II is non-insulin dependent and Obesity (syndrome X) is an


important risk factor for development of type II diabetes mellitus
and insulin resistance. Target tissues for insulin do not respond to
circulating insulin and there is a decrease in insulin secretion with
time.
i. In the early stages there is a hypersecretion of insulin and
target tissues are not responding to insulin. In the later
stages there is decreased secretion of insulin from the cells (-cell fatigue) combined with insulin resistance.
3. Correlate the presenting features of diabetes and biochemical
basis for the clinical features
a. Polyuria- You pee more because you have a lot of glucose. Not all
the glucose is reabsorbed in the kidney so when you pee the
glucose takes water with it.
b. Polydipsia- Since you are peeing more and the glucose in the pee is
taking water you feel more thirsty
c. Polyphagia- You are hungry because you have insulin resistance and
leptin resistance is usually coupled with this (I think this is right)
4. Summarize the complications of diabetes and correlate to
biochemical and metabolic changes
a. Acute complications
i. Ketoacidosis (Type I)- Uncontrolled lipolysis of adipose
tissue because of no insulin. Results in increased FFA to the
liver and increased -oxidation. The results in the formation
of acetyl CoA for ketone body synthesis. Ketogenesis >>>
Peripheral utilization of ketone bodies. Ketone bodies (Weak
acids) in blood. You get a fruity odor because some ketone
bodies spontaneously get converted to acetone.
ii. Hyperosmolar non-ketotic coma- Coma is due to
Hyperglycemia and because glucose is being excreted it is
taking water with it. This causes neuronal dehydration.
b. Chronic
i. Microvascular- Neuropathy, nephropathy and retinopathy
are microvascular complications. These are tissues which do
not require insulin for glucose entry (retina, nervous tissue,
lens). Sorbitol formation in the lens, nerve and kidney
results in cell swelling due to water retention. Glycation of
proteins (AGEs) of the basement membrane contributes to
nephropathy. You also get foot ulcers and increased
excretion of albumin in urine in the initial stages. **AGE is
advanced glycation of proteins.
ii. Macrovascular- You have hyper TAGs. There is increased
chylomicrons and VLDL in the circulation. This is due to
decreased action of lipoprotein lipase present in the
endothelium of blood vessels (LPL needs insulin for optimal

71

activity). Insulin resistance increases the risk of


cardiovascular disease in diabetes.
5. Acute complications of diabetes mellitus:
a. Sequence of metabolic changes resulting in diabetic
ketoacidosis (type I)- You need insulin to stop HS lipase from
releasing FFA and the liver from converting them to ketone bodies.
Because type I has no insulin you cannot regulate any of these
processes resulting in an increase in ketone bodies causing
metabolic acidosis.
b. Compare and contrast biochemical and laboratory findings
in hyperosmolar non-ketotic coma and diabetic ketoacidosisHyperosmolar is found in Type II while diabetic ketoacidosis is found
in Type I. Ketosis is not prominent as in type I diabetics because you
have insulin that can regulate HS lipase.
c. Relate the occurrence of hypoglycemia in diabetic patients
on insulin- Sometimes you can give them too much insulin and
that results in a depletion of serum glucose.
6. Evaluate the significance of the laboratory tests for diabetes
a. Tests for diagnosis of diabetes: Fasting blood glucose,
postprandial blood glucose, role of GTT
i. Fasting blood glucose > 126 mg/dL
ii. Elevated postprandial blood glucose (blood glucose
obtained 2 hours after a meal)
iii. Oral glucose tolerance tests is performed on an individual
after an overnight fast. Subject is given a known load of
glucose orally. Blood glucose is estimated at the beginning
of the test and every half hour for up to 2 hours. In a normal
person the 2 hr blood glucose is less than 200 mg/dL.
Diabetic patient has decreased tolerance to glucose.
b. Tests for long term management:
i. Tests to assess glycemic control (Fasting blood glucose,
postprandial blood glucose, glycated hemoglobin)
ii. Tests for lipid profile (Fasting lipid profile, serum cholesterol,
serum TAGs, LDL levels, HDL levels)
iii. Tests to assess renal function- (measurement of albumin
excretion in the urine (microalbuminuria)
Lecture: Hypoglycemia
1. Summarize the hormonal regulation of blood glucose
a. At low blood glucose levels several systems are used to normalize
blood glucose. The pituitary gland and ACTH, the autonomic
nervous system or directly via the low serum blood glucose levels
acting on the -cells of the pancreas
2. Analyze the effects of hypoglycemia
a. Explain the release of counter-regulatory hormones
72

i.

Response to low blood glucose levels leads to a release of


catecholamines
and
cortisol.
The
cortisol
promotes
gluconeogenesis while the norepinephrine and epinephrine
stimulate glycogenolysis. Glucagon from the -cells of the
pancreas stimulates both glycogenolysis and gluconeogenesis.
b. Discriminate
the
manifestations
of
hypoglycemia
Adrenergic, neuroglycopenic symptoms
i. Hypoglycemia is characterized by a blood glucose level less
than 40mg/dL. Severe hypoglycemia is characterized by blood
glucose less than 20 mg/dL. Adrenergic symptoms start at
glucose blood levels of about 55 mg/dL and they include
anxiety palpitations, tremor and sweating. Neuroglycopenia
symptoms start at glucose blood levels at about 50 mg/dL and
they include headache, confusion, slurred speech, seizures and
even coma and death
ii. The
types
of
hypoglycemia
include
insulin-induced
hypoglycemia in patients treated with insulin or in patients
with
insulinoma.
Postprandial
hypoglycemia
due
to
exaggerated insulin release in some individuals. Fasting
hypoglycemia signals a serious medical problem, often genetic
defects. Lastly you have hypoglycemia due to alcohol
intoxication (High NADH ratio consumes gluconeogenesis
intermediates cant form OAA from malate).
3. Distinguish and discuss the mechanisms of causation of
hypoglycemia and the differences between them
a. Fasting hypoglycemia- found at high ethanol levels and
especially in undernourished fasted or dehydrated individuals that
consume ethanol (high NADH ratio interferes with usage of
gluconeogenesis precursors). Lactic acidosis also occurs because
pyruvate lactate and leads to gout as uric acid and lactate build
up in kidney. Also found in genetic defects of beta oxidation (MCAD
deficiency, carnitine shuttle deficiency and CPT I deficiency). CPT II
defect does not cause hypoglycemia because it is found in muscle
and muscle cells cannot do gluconeogenesis. Lastly you can find
hypoglycemia in Von Gierkes disease (very severe) because you
dont have glucose 6-phosphatase so glucose cannot be released
from the cell.
b. Reactive
(postprandial)
hypoglycemiaPostprandial
hypoglycemia is quite common and occurs in individuals with
exaggerated insulin release after a meal. This is not a severe
condition and leads to transient hypoglycemia with mild adrenergic
symptoms. Plasma glucose levels return to normal without
treatment and preventable by frequent small meals
c. Alcohol induced hypoglycemia- Ethanol-induced hypoglycemia is
found in individuals after excessive intake of ethanol. This is often
found in undernourished, fasted or dehydrated individuals. High
73

levels of ethanol in liver cytosol lead to fasting hypoglycemia when


the glycogen stores are depleted and gluconeogenesis is the source
for release of glucose into the blood. The high cytosolic NADH/NAD +
ratio prevents the normal usage of lactate, amino acids and glycerol
for gluconeogenesis. NADH is formed in cytosol by alcohol
dehydrogenase and also by cytosolic acetaldehyde dehydrogenase.
d. Factitious hypoglycemia- (Needs to be investigated) It is a
mental/personal disease where the patient tries to acquire
hypoglycemia due to injection of insulin or usage of specific medical
drugs. You have to determine serum insulin, C-peptide, proinsulin
and sulfonylurea. If only insulin is high and C-peptide and proinsulin
are not elevated, then insulin was injected. If insulin, C-peptide and
proinsulin are high then endogenous overproduction of insulin took
place. Sulfonylura is an antidiabetic drug used for treatment of
diabetes mellitus Type 2. This drug stimulates insulin secretion.
e. Hypoglycemia of insulinoma- This type of hypoglycemia is
grouped as insulin-induced hypoglycemia. Severe hypoglycemia can
occur during fasting in patients with insulinoma. The tumor
produces high levels of insulin which blocks the action of the insulin
counter-acting hormones. Patients with insulinoma have high blood
glucose levels of insulin, C-peptide and proinsulin (Endogenous).
4. Justify the significance of estimation of serum insulin, C-peptide,
proinsulin in the various types of hypoglycemia
5. Integrate knowledge from previously studied metabolic pathways
to identify some of the causes of hypoglycemia in infancy and
childhood (von Gierkes disease, MCAD and carnitine deficiency)
Kind of described abovethis is in Trotz questions but both all of them
cause hypoglycemia. MCAD and Carnitine cause hypoketonemia in
addition to hypoglycemia.
Lecture: Molecular mechanisms in inherited diseases
1. Describe the molecular and biochemical basis for symptoms,
diagnostic tests and treatment of
2. Cystic Fibrosis
a. Autosomal recessive disorder in Caucasian children. Due to a
mutation in the gene for Cystic Fibrosis Transmembrane
Conductance Regulator (CFTR), a membrane protein that acts as a
chloride channel. Belongs to the group of an ATP binding cassette
transporter. Phosphorylation of the cytoplasmic regulatory domain
by PKA activates the channel providing regulated Cl- and fluid
secretion. In the ducts it traps the Cl- in the cell creating a more
viscous mucosa. In the sweat glands the CFTR is involved in the
reabsorption of NaCl. Very little NaCl is reabsorbed resulting in a
high salt content in sweat.

74

b. Symptoms associated with CF include malabsorption, abnormal


sweat electrolytes, chronic pancreatitis, lung abscess, chronic
bronchitis and honeycomb lung.
c. The mutant CFTR doesnt transport Cl- into the airway lumen and as
a result Na and H2O content of the luminal secretions is low
resulting in thickened and viscid mucus secretions. More prone for
bacterial infections. Respiratory infections are the most common
cause of mortality and morbidity in patients with cystic fibrosis.
d. Exocrine Pancreas deficiency because loss of CFTR leads to thicker
acinar secretions within the duct lumen leading to obstruction and
tissue destruction. Fibrotic tissue and fat replace the pancreatic
parenchyma hence the name cystic fibrosis. Leads to maldigestion
of nutrients and to excretion of fat in the stool (steatorrhea). More
than 95% of males with CF are infertile because they lack a vas
deferens a phenotype known as congenital bilateral absence of the
vas deferens (CBAVD).
e. Genetic Basis- Results from mutations at the single gene locus on
the long arm of chromosome 7. Most common CF disorder is of
F508. It is a 3-bp deletion that eliminates the phenylalanine
residue of CFTR at position 508. ASO test is useful if the mutation is
known,
3. Sickle cell anemia
a. Point mutation in the -globin chain of hemoglobin. Glutamic acid is
replaced by valine at the sixth position of hemoglobin. Replacement
of valine in sickle cell disease hemoglobin tends to aggregate to
form long filament like structures. This aggregation results in
distortion of the structure of RBC-sickling.
b. The sickled and distorted cells are periodically removed by the
spleen hence the patients have anemia. Spleen may also be
enlarged (Splenomegaly). Since RBCs are being destroyed you get
anemia and pre-hepatic jaundice. Characterized by high levels of
unconjugated bilirubin. Urine is normal color and due to the
excessive loss of conjugated bilirubin in the bile people may
develop pigmented gall stones.
c. Biochemical diagnosis- Hemoglobin electrophoresis: HbS (Valine)
has fewer number of negative charges than HbA (Glutamate). HbS
moves slower than HbA toward the anode (+). ASO test (Dot Blot
test) using this assay it is possible to identify heterozygous carriers.
4. Duchenne Muscular Dystrophy and Becker Muscular Dystrophya. Both the disorders are due to mutations in the dystrophin gene.
They dystrophin gene is the largest gene located on the Xchromosome. DMD is due to the almost complete absence of
functional dystrophin. BMD is due to the production of abnormal
dystrophin or less amounts of dystrophin.
b. Duchenne Muscular Dystrophy- due to mutations in a gene on the X
chromosome that prevent the production of dystrophin, a muscle
75

protein. X-linked recessive disease and more common in males.


Most patients die in their early 20s due to breathing and heart
problems.
c. Becker Muscular Dystrophy- BMD is a milder form DMD. Symptoms
are similar to the DMD but the onset is later and the course of the
disease is milder. Both DMD and BMD result from a mutation in the
dystrophin gene. The DMD deletions can however be distinguished
from deletions causing BMD, when viewed at the codon level.
Almost all deletions causing DMD involve frameshift mutations,
abolishing translation of dystrophin protein. Many of the mutations
also involve large deletion of exons. BMD deletions are in frame
hence some dystrophin although truncated is translated.
5. Identify the differences between Duchenne muscular dystrophy
and Becker muscular dystrophy with regard to severity of the
disorder, type of mutations, alterations in dystrophin
a. DMD- Because of weakened muscles they have a hard time from
rising from the floor Gowers maneuver. Dystrophin is encoded by
the largest gene described to date. Gene expressed mainly in
smooth, cardiac, and skeletal muscle, with lower levels in the brain.
Dystrophin anchors the cytoskeleton of muscle cells to the EC
matrix. Dystrophin links actin filaments to transmembrane proteins
of the muscle cell plasma membrane. Functional loss of dystrophin
leads to oxidative cellular injury and myonecrosis.
b. Western blot has smaller protein size, and reduced quantity of
dystrophin protein in BMD. Complete absence of dystrophin in DMD.
Reduced quantity of complete absence of dystrophin in DMD. Serum
Creatine kinase (MM), levels are elevated in patients with muscular
dystrophy (indicative of muscle damage). Females are carriers and
they usually have higher levels of CK-MM in serum. Multiplex PCR
analysis is usually used to identify the specific mutation.

76

You might also like